Download as doc, pdf, or txt
Download as doc, pdf, or txt
You are on page 1of 68

Nursing: A Concept-Based Approach to Learning, 2e (Pearson)

Module 12 Metabolism

The Concept of Metabolism

1) A client is experiencing health problems related to alterations in adrenal medulla function. On


which areas should the nurse focus when assessing this client?
Select all that apply.
A) Heart rate
B) Weight
C) Respiratory rate
D) Skin integrity
E) Blood pressure
Answer: A, B, C, E
Explanation: A) The adrenal medulla secretes the catecholamines of epinephrine and
norepinephrine. These hormones stimulate the heart, increase metabolism, increase respiration,
and constrict blood vessels. The nurse should focus the assessment on heart rate, weight,
respiratory rate, and blood pressure. Skin integrity is not influenced by these hormones.
B) The adrenal medulla secretes the catecholamines of epinephrine and norepinephrine. These
hormones stimulate the heart, increase metabolism, increase respiration, and constrict blood
vessels. The nurse should focus the assessment on heart rate, weight, respiratory rate, and blood
pressure. Skin integrity is not influenced by these hormones.
C) The adrenal medulla secretes the catecholamines of epinephrine and norepinephrine. These
hormones stimulate the heart, increase metabolism, increase respiration, and constrict blood
vessels. The nurse should focus the assessment on heart rate, weight, respiratory rate, and blood
pressure. Skin integrity is not influenced by these hormones.
D) The adrenal medulla secretes the catecholamines of epinephrine and norepinephrine. These
hormones stimulate the heart, increase metabolism, increase respiration, and constrict blood
vessels. The nurse should focus the assessment on heart rate, weight, respiratory rate, and blood
pressure. Skin integrity is not influenced by these hormones.
E) The adrenal medulla secretes the catecholamines of epinephrine and norepinephrine. These
hormones stimulate the heart, increase metabolism, increase respiration, and constrict blood
vessels. The nurse should focus the assessment on heart rate, weight, respiratory rate, and blood
pressure. Skin integrity is not influenced by these hormones.
Page Ref: 726
Cognitive Level: Creating
Client Need: Health Promotion and Maintenance
Nursing Process: Assessment
Learning Outcome: 1. Summarize the structure and physiology of the body related to
metabolism.

1
Copyright © 2015 Pearson Education, Inc.
2) The nurse is teaching an in-service about metabolic disorders. Which person is at the greatest
risk for malnutrition as a result of hypermetabolism?
A) A client with chronic obstructive pulmonary disease
B) A client with osteoporosis
C) A client who is a vegetarian
D) A client who has dysphagia
Answer: A
Explanation: A) The client with chronic obstructive pulmonary disease often is malnourished
because of the increased caloric need associated with breathing efforts. The client with dysphagia
is at risk for malnutrition because of the inability to eat adequate amounts of foods. Patients with
osteoporosis do not have higher metabolic rates. There is no connection between
hypermetabolism and vegetarianism.
B) The client with chronic obstructive pulmonary disease often is malnourished because of the
increased caloric need associated with breathing efforts. The client with dysphagia is at risk for
malnutrition because of the inability to eat adequate amounts of foods. Patients with osteoporosis
do not have higher metabolic rates. There is no connection between hypermetabolism and
vegetarianism.
C) The client with chronic obstructive pulmonary disease often is malnourished because of the
increased caloric need associated with breathing efforts. The client with dysphagia is at risk for
malnutrition because of the inability to eat adequate amounts of foods. Patients with osteoporosis
do not have higher metabolic rates. There is no connection between hypermetabolism and
vegetarianism.
D) The client with chronic obstructive pulmonary disease often is malnourished because of the
increased caloric need associated with breathing efforts. The client with dysphagia is at risk for
malnutrition because of the inability to eat adequate amounts of foods. Patients with osteoporosis
do not have higher metabolic rates. There is no connection between hypermetabolism and
vegetarianism.
Page Ref: 730
Cognitive Level: Analyzing
Client Need: Physiological Integrity
Nursing Process: Assessment
Learning Outcome: 2. Examine the relationship between metabolism and other
concepts/systems.

2
Copyright © 2015 Pearson Education, Inc.
3) The nurse is assessing the vital signs of a client experiencing hypoparathyroidism. While
monitoring the blood pressure, the nurse notes the client's hand begins to spasm. How should the
nurse document this assessment finding?
A) Trousseau sign
B) Chvostek sign
C) Turner's sign
D) Cullen's sign
Answer: A
Explanation: A) Trousseau sign is elicited when placing a blood pressure cuff on the arm; when
the cuff is inflated, the client experiences carpal spasms. Chvostek sign is elicited by tapping on
the face in front of the ear and observing for contractions of the muscle. Turner's sign and
Cullen's sign are both associated with retroperitoneal bleeding.
B) Trousseau sign is elicited when placing a blood pressure cuff on the arm; when the cuff is
inflated, the client experiences carpal spasms. Chvostek sign is elicited by tapping on the face in
front of the ear and observing for contractions of the muscle. Turner's sign and Cullen's sign are
both associated with retroperitoneal bleeding.
C) Trousseau sign is elicited when placing a blood pressure cuff on the arm; when the cuff is
inflated, the client experiences carpal spasms. Chvostek sign is elicited by tapping on the face in
front of the ear and observing for contractions of the muscle. Turner's sign and Cullen's sign are
both associated with retroperitoneal bleeding.
D) Trousseau sign is elicited when placing a blood pressure cuff on the arm; when the cuff is
inflated, the client experiences carpal spasms. Chvostek sign is elicited by tapping on the face in
front of the ear and observing for contractions of the muscle. Turner's sign and Cullen's sign are
both associated with retroperitoneal bleeding.
Page Ref: 737
Cognitive Level: Applying
Client Need: Physiological Integrity
Nursing Process: Assessment
Learning Outcome: 3. Identify commonly occurring alterations in metabolism and their related
therapies.

3
Copyright © 2015 Pearson Education, Inc.
4) While performing an endocrine assessment on a client suspected of having Cushing disease,
the nurse asks the female client if she has experienced recent weight changes. Which endocrine
systems is the nurse assessing?
Select all that apply.
A) Gonads
B) Pituitary gland
C) Thyroid gland
D) Adrenal gland
E) Parathyroid gland
Answer: B, C, D
Explanation: A) In adrenal, thyroid, and pituitary disease, the client's weight changes might
provide information as to the endocrine disorder the client is experiencing. The client might gain
weight with an adrenal disorder such as Cushing disease, or thyroid disease such as
hypothyroidism or hyperthyroidism. The pituitary gland controls ADH, which influences water
absorption by the renal tubules. The parathyroid gland regulates calcium and phosphorous. The
gonads influence estrogen and androgens.
B) In adrenal, thyroid, and pituitary disease, the client's weight changes might provide
information as to the endocrine disorder the client is experiencing. The client might gain weight
with an adrenal disorder such as Cushing disease, or thyroid disease such as hypothyroidism or
hyperthyroidism. The pituitary gland controls ADH, which influences water absorption by the
renal tubules. The parathyroid gland regulates calcium and phosphorous. The gonads influence
estrogen and androgens.
C) In adrenal, thyroid, and pituitary disease, the client's weight changes might provide
information as to the endocrine disorder the client is experiencing. The client might gain weight
with an adrenal disorder such as Cushing disease, or thyroid disease such as hypothyroidism or
hyperthyroidism. The pituitary gland controls ADH, which influences water absorption by the
renal tubules. The parathyroid gland regulates calcium and phosphorous. The gonads influence
estrogen and androgens.
D) In adrenal, thyroid, and pituitary disease, the client's weight changes might provide
information as to the endocrine disorder the client is experiencing. The client might gain weight
with an adrenal disorder such as Cushing disease, or thyroid disease such as hypothyroidism or
hyperthyroidism. The pituitary gland controls ADH, which influences water absorption by the
renal tubules. The parathyroid gland regulates calcium and phosphorous. The gonads influence
estrogen and androgens.
E) In adrenal, thyroid, and pituitary disease, the client's weight changes might provide
information as to the endocrine disorder the client is experiencing. The client might gain weight
with an adrenal disorder such as Cushing disease, or thyroid disease such as hypothyroidism or
hyperthyroidism. The pituitary gland controls ADH, which influences water absorption by the
renal tubules. The parathyroid gland regulates calcium and phosphorous. The gonads influence
estrogen and androgens.
Page Ref: 733
Cognitive Level: Analyzing
Client Need: Health Promotion and Maintenance
Nursing Process: Assessment
Learning Outcome: 4. Differentiate common assessment procedures used to examine
metabolism across the life span.

4
Copyright © 2015 Pearson Education, Inc.
5) The nurse is reviewing the laboratory test results for a client with an endocrine disorder. For
which tests should the nurse expect to have current values on the medical record?
Select all that apply.
A) Prothrombin time
B) Albumin
C) Ammonia level
D) Liver functions studies
E) Hemoglobin and hematocrit
Answer: B, D
Explanation: A) Specific diagnostic tests to assess the structure and function of the glands of the
endocrine system include serum albumin liver function studies such as AST, ALT, SGOT, and
LDH. Prothrombin time, ammonia level, and hemoglobin and hematocrit are not tests to
specifically assess the structure and function of the endocrine glands.
B) Specific diagnostic tests to assess the structure and function of the glands of the endocrine
system include serum albumin liver function studies such as AST, ALT, SGOT, and LDH.
Prothrombin time, ammonia level, and hemoglobin and hematocrit are not tests to specifically
assess the structure and function of the endocrine glands.
C) Specific diagnostic tests to assess the structure and function of the glands of the endocrine
system include serum albumin liver function studies such as AST, ALT, SGOT, and LDH.
Prothrombin time, ammonia level, and hemoglobin and hematocrit are not tests to specifically
assess the structure and function of the endocrine glands.
D) Specific diagnostic tests to assess the structure and function of the glands of the endocrine
system include serum albumin liver function studies such as AST, ALT, SGOT, and LDH.
Prothrombin time, ammonia level, and hemoglobin and hematocrit are not tests to specifically
assess the structure and function of the endocrine glands.
E) Specific diagnostic tests to assess the structure and function of the glands of the endocrine
system include serum albumin liver function studies such as AST, ALT, SGOT, and LDH.
Prothrombin time, ammonia level, and hemoglobin and hematocrit are not tests to specifically
assess the structure and function of the endocrine glands.
Page Ref: 732
Cognitive Level: Applying
Client Need: Physiological Integrity
Nursing Process: Assessment
Learning Outcome: 5. Describe diagnostic and laboratory tests to determine an individual's
metabolic status.

5
Copyright © 2015 Pearson Education, Inc.
6) The client with diabetes mellitus reports having difficulty cutting his toenails because they are
thick and ingrown. What should the nurse recommend to this client?
A) Make an appointment with a podiatrist.
B) Offer to file the tops of the nails to reduce thickness after cutting.
C) Cut the nails straight across with a clipper after the bath.
D) Make an appointment with a nail shop for a pedicure.
Answer: A
Explanation: A) The toenails of the client with diabetes require close care. If the nails are thick
or ingrown, they require the attention of a podiatrist. Cutting the nails across after the bath is
correct for toenails that do not demonstrate the complications listed. The client with diabetes is at
an increased risk for infection and should avoid situations in which this risk is increased, such as
the nail shop pedicure. The nurse should not cut the client's toenails.
B) The toenails of the client with diabetes require close care. If the nails are thick or ingrown,
they require the attention of a podiatrist. Cutting the nails across after the bath is correct for
toenails that do not demonstrate the complications listed. The client with diabetes is at an
increased risk for infection and should avoid situations in which this risk is increased, such as the
nail shop pedicure. The nurse should not cut the client's toenails.
C) The toenails of the client with diabetes require close care. If the nails are thick or ingrown,
they require the attention of a podiatrist. Cutting the nails across after the bath is correct for
toenails that do not demonstrate the complications listed. The client with diabetes is at an
increased risk for infection and should avoid situations in which this risk is increased, such as the
nail shop pedicure. The nurse should not cut the client's toenails.
D) The toenails of the client with diabetes require close care. If the nails are thick or ingrown,
they require the attention of a podiatrist. Cutting the nails across after the bath is correct for
toenails that do not demonstrate the complications listed. The client with diabetes is at an
increased risk for infection and should avoid situations in which this risk is increased, such as the
nail shop pedicure. The nurse should not cut the client's toenails.
Page Ref: 734
Cognitive Level: Applying
Client Need: Health Promotion and Maintenance
Nursing Process: Implementation
Learning Outcome: 6. Explain management of metabolic health and prevention of metabolic
disorders.

6
Copyright © 2015 Pearson Education, Inc.
7)

An older client is diagnosed with disorders of fat metabolism, reduced absorption of fat-soluble
vitamins, and slightly elevated blood glucose level. When caring for this client, on which
endocrine organ should the nurse focus interventions?
A) Pituitary
B) Thyroid
C) Pancreas
D) Adrenal medulla
Answer: C
Explanation: A) Normal age-related changes in pancreatic function include decreased
production of lipase, which causes less fat absorption and decreased absorption of fat-soluble
vitamins. The pancreas also has decreased insulin release and decreased sensitivity to circulating
insulin, which can cause an increase in blood glucose levels in the older adult who does not have
diabetes. There is no evidence to support focusing on the pituitary, thyroid, or adrenal medulla.
B) Normal age-related changes in pancreatic function include decreased production of lipase,
which causes less fat absorption and decreased absorption of fat-soluble vitamins. The pancreas
also has decreased insulin release and decreased sensitivity to circulating insulin, which can
cause an increase in blood glucose levels in the older adult who does not have diabetes. There is
no evidence to support focusing on the pituitary, thyroid, or adrenal medulla.
C) Normal age-related changes in pancreatic function include decreased production of lipase,
which causes less fat absorption and decreased absorption of fat-soluble vitamins. The pancreas
also has decreased insulin release and decreased sensitivity to circulating insulin, which can
cause an increase in blood glucose levels in the older adult who does not have diabetes. There is
no evidence to support focusing on the pituitary, thyroid, or adrenal medulla.

7
Copyright © 2015 Pearson Education, Inc.
D) Normal age-related changes in pancreatic function include decreased production of lipase,
which causes less fat absorption and decreased absorption of fat-soluble vitamins. The pancreas
also has decreased insulin release and decreased sensitivity to circulating insulin, which can
cause an increase in blood glucose levels in the older adult who does not have diabetes. There is
no evidence to support focusing on the pituitary, thyroid, or adrenal medulla.
Page Ref: 727
Cognitive Level: Analyzing
Client Need: Health Promotion and Maintenance
Nursing Process: Implementation
Learning Outcome: 7. Demonstrate the nursing process in providing culturally competent and
caring interventions across the life span for individuals with common alterations in metabolism.

8) The nurse is caring for a client who has been prescribed calcitonin–human (Cibacalcin) nasal
spray. What should the nurse include in the plan of care for this client?
A) Take 1 hour before meals or 2 hours after meals.
B) Alternate nostrils used daily.
C) Take on an empty stomach in the morning with water.
D) Remain in an upright position for 30 minutes after taking.
Answer: B
Explanation: A) When the client is prescribed a hormone supplement to be provided through a
nasal spray, the nurse should alternate the nostrils used daily. This medication is not provided
through the oral route, so it is not necessary to provide 1 hour before or 2 hours after meals. This
medication does not need to be provided on an empty stomach first thing in the morning. The
client does not need to remain in an upright position for 30 minutes after taking this medication.
B) When the client is prescribed a hormone supplement to be provided through a nasal spray, the
nurse should alternate the nostrils used daily. This medication is not provided through the oral
route, so it is not necessary to provide 1 hour before or 2 hours after meals. This medication does
not need to be provided on an empty stomach first thing in the morning. The client does not need
to remain in an upright position for 30 minutes after taking this medication.
C) When the client is prescribed a hormone supplement to be provided through a nasal spray, the
nurse should alternate the nostrils used daily. This medication is not provided through the oral
route, so it is not necessary to provide 1 hour before or 2 hours after meals. This medication does
not need to be provided on an empty stomach first thing in the morning. The client does not need
to remain in an upright position for 30 minutes after taking this medication.
D) When the client is prescribed a hormone supplement to be provided through a nasal spray, the
nurse should alternate the nostrils used daily. This medication is not provided through the oral
route, so it is not necessary to provide 1 hour before or 2 hours after meals. This medication does
not need to be provided on an empty stomach first thing in the morning. The client does not need
to remain in an upright position for 30 minutes after taking this medication.
Page Ref: 738
Cognitive Level: Applying
Client Need: Physiological Integrity
Nursing Process: Planning
Learning Outcome: 8. Compare and contrast common independent and collaborative
interventions for clients with alterations in metabolism.

8
Copyright © 2015 Pearson Education, Inc.
Exemplar 12.1 Diabetes

1) The nurse is completing an assessment interview with a client being seen for a yearly physical
examination. Which client statement would indicate a possible diagnosis of diabetes?
A) "I'm slightly winded when I walk up a flight of stairs, but it passes quickly."
B) "I feel a bit tired by mid-afternoon and take a 30-minute nap most days."
C) "I sometimes have muscle aches in my upper legs at night."
D) "I've been experiencing increased thirst during the past several months."
Answer: D
Explanation: A) Excessive thirst can be associated with high glucose levels and may be a
symptom of undiagnosed diabetes mellitus. Fatigue that responds to a short nap, having some
muscle aches at night, and being slightly short of breath after walking up a flight of stairs with a
quick recovery may be within the normal functioning of a healthy older client.
B) Excessive thirst can be associated with high glucose levels and may be a symptom of
undiagnosed diabetes mellitus. Fatigue that responds to a short nap, having some muscle aches at
night, and being slightly short of breath after walking up a flight of stairs with a quick recovery
may be within the normal functioning of a healthy older client.
C) Excessive thirst can be associated with high glucose levels and may be a symptom of
undiagnosed diabetes mellitus. Fatigue that responds to a short nap, having some muscle aches at
night, and being slightly short of breath after walking up a flight of stairs with a quick recovery
may be within the normal functioning of a healthy older client.
D) Excessive thirst can be associated with high glucose levels and may be a symptom of
undiagnosed diabetes mellitus. Fatigue that responds to a short nap, having some muscle aches at
night, and being slightly short of breath after walking up a flight of stairs with a quick recovery
may be within the normal functioning of a healthy older client.
Page Ref: 742
Cognitive Level: Analyzing
Client Need: Physiological Integrity
Nursing Process: Assessment
Learning Outcome: 1. Describe the pathophysiology, etiology, clinical manifestations, and
direct and indirect causes of diabetes mellitus.

9
Copyright © 2015 Pearson Education, Inc.
2) The nurse, teaching a class to a group of community members about the importance of weight
loss in decreasing the risk of type 2 diabetes mellitus, is asked why weight loss reduces the risk
associated with the development of this health problem. Which response by the nurse is most
correct?
A) "Excess body weight impairs the body's release of insulin."
B) "The amount of food taken in by those who are overweight requires more insulin to
adequately metabolize them, resulting in diabetes."
C) "The physical inactivity associated with obesity causes a reduced ability by the body to
produce insulin."
D) "Thin people are less likely to become diabetic."
Answer: A
Explanation: A) Beta cells of the body release insulin. Their actions are hindered as the amount
of adipose tissue in the body increases. The amount of food taken in is not the issue as much as
the excess body weight. The body does require more insulin with a greater food intake, but that
does not necessarily result in diabetes. While obesity is a risk factor for the development of
diabetes, this does not meet the question posed by the client. Inactivity is directly linked to
obesity, but it does not present a direct tie to the production of insulin.
B) Beta cells of the body release insulin. Their actions are hindered as the amount of adipose
tissue in the body increases. The amount of food taken in is not the issue as much as the excess
body weight. The body does require more insulin with a greater food intake, but that does not
necessarily result in diabetes. While obesity is a risk factor for the development of diabetes, this
does not meet the question posed by the client. Inactivity is directly linked to obesity, but it does
not present a direct tie to the production of insulin.
C) Beta cells of the body release insulin. Their actions are hindered as the amount of adipose
tissue in the body increases. The amount of food taken in is not the issue as much as the excess
body weight. The body does require more insulin with a greater food intake, but that does not
necessarily result in diabetes. While obesity is a risk factor for the development of diabetes, this
does not meet the question posed by the client. Inactivity is directly linked to obesity, but it does
not present a direct tie to the production of insulin.
D) Beta cells of the body release insulin. Their actions are hindered as the amount of adipose
tissue in the body increases. The amount of food taken in is not the issue as much as the excess
body weight. The body does require more insulin with a greater food intake, but that does not
necessarily result in diabetes. While obesity is a risk factor for the development of diabetes, this
does not meet the question posed by the client. Inactivity is directly linked to obesity, but it does
not present a direct tie to the production of insulin.
Page Ref: 742-743
Cognitive Level: Analyzing
Client Need: Health Promotion and Maintenance
Nursing Process: Implementation
Learning Outcome: 2. Identify risk factors and prevention methods associated with diabetes
mellitus.

10
Copyright © 2015 Pearson Education, Inc.
3) A 58-year-old client who is newly diagnosed with type 2 diabetes has smoked for 30 years.
When teaching the client on ways to optimize health outcomes, what should the nurse explain
about the effects of smoking and diabetes?
A) Smoking is a major factor in the development of diabetic neuropathy.
B) Smoking increases insulin resistance.
C) Smoking accelerates arteriosclerotic changes in blood vessels.
D) Smoking promotes weight gain.
Answer: C
Explanation: A) Smoking is especially unhealthy for diabetic clients because smoking
accelerates the arteriosclerotic effects that occur in blood vessels from elevated levels of blood
glucose. Smoking is not associated with weight gain; in fact, people use weight gain as an excuse
not to quit smoking. Poor glycemic control in diabetics is associated with the development of
complications including diabetic neuropathy. Smoking does not affect insulin resistance.
B) Smoking is especially unhealthy for diabetic clients because smoking accelerates the
arteriosclerotic effects that occur in blood vessels from elevated levels of blood glucose.
Smoking is not associated with weight gain; in fact, people use weight gain as an excuse not to
quit smoking. Poor glycemic control in diabetics is associated with the development of
complications including diabetic neuropathy. Smoking does not affect insulin resistance.
C) Smoking is especially unhealthy for diabetic clients because smoking accelerates the
arteriosclerotic effects that occur in blood vessels from elevated levels of blood glucose.
Smoking is not associated with weight gain; in fact, people use weight gain as an excuse not to
quit smoking. Poor glycemic control in diabetics is associated with the development of
complications including diabetic neuropathy. Smoking does not affect insulin resistance.
D) Smoking is especially unhealthy for diabetic clients because smoking accelerates the
arteriosclerotic effects that occur in blood vessels from elevated levels of blood glucose.
Smoking is not associated with weight gain; in fact, people use weight gain as an excuse not to
quit smoking. Poor glycemic control in diabetics is associated with the development of
complications including diabetic neuropathy. Smoking does not affect insulin resistance.
Page Ref: 748
Cognitive Level: Applying
Client Need: Health Promotion and Maintenance
Nursing Process: Implementation
Learning Outcome: 3. Illustrate the nursing process in providing culturally competent care
across the life span for individuals with diabetes mellitus.

11
Copyright © 2015 Pearson Education, Inc.
4) The nurse is planning care for an 86-year-old client with type 2 diabetes mellitus. Which
nursing diagnosis would be most appropriate for this client?
A) Risk for Falls
B) Risk for Infection
C) Ineffective Tissue Perfusion: Cardiac
D) Impaired Tissue Integrity
Answer: B
Explanation: A) A client with diabetes mellitus is at risk for infection. No other information is
given in the question with regard to risk for falls, ineffective tissue perfusion, or impaired tissue
integrity as potential nursing diagnoses.
B) A client with diabetes mellitus is at risk for infection. No other information is given in the
question with regard to risk for falls, ineffective tissue perfusion, or impaired tissue integrity as
potential nursing diagnoses.
C) A client with diabetes mellitus is at risk for infection. No other information is given in the
question with regard to risk for falls, ineffective tissue perfusion, or impaired tissue integrity as
potential nursing diagnoses.
D) A client with diabetes mellitus is at risk for infection. No other information is given in the
question with regard to risk for falls, ineffective tissue perfusion, or impaired tissue integrity as
potential nursing diagnoses.
Page Ref: 760
Cognitive Level: Applying
Client Need: Physiological Integrity
Nursing Process: Planning
Learning Outcome: 4. Formulate priority nursing diagnoses appropriate for an individual with
diabetes mellitus.

12
Copyright © 2015 Pearson Education, Inc.
5) The nurse is preparing to teach a client who is newly diagnosed with type 1 diabetes mellitus
on the preferred area to self-inject insulin. On which area should the nurse focus based upon
insulin absorption rates?
A) Deltoid
B) Thigh
C) Hip
D) Abdomen
Answer: D
Explanation: A) The rate of absorption and peak of action of insulin differ according to the site.
The site that allows the most rapid absorption is the abdomen, followed by the deltoid muscle,
then the thigh, and then the hip. Because of the rapid absorption, the abdomen is the
recommended site.
B) The rate of absorption and peak of action of insulin differ according to the site. The site that
allows the most rapid absorption is the abdomen, followed by the deltoid muscle, then the thigh,
and then the hip. Because of the rapid absorption, the abdomen is the recommended site.
C) The rate of absorption and peak of action of insulin differ according to the site. The site that
allows the most rapid absorption is the abdomen, followed by the deltoid muscle, then the thigh,
and then the hip. Because of the rapid absorption, the abdomen is the recommended site.
D) The rate of absorption and peak of action of insulin differ according to the site. The site that
allows the most rapid absorption is the abdomen, followed by the deltoid muscle, then the thigh,
and then the hip. Because of the rapid absorption, the abdomen is the recommended site.
Page Ref: 756
Cognitive Level: Analyzing
Client Need: Physiological Integrity
Nursing Process: Planning
Learning Outcome: 6. Plan evidence-based care for an individual with diabetes mellitus and his
or her family in collaboration with other members of the healthcare team.

13
Copyright © 2015 Pearson Education, Inc.
6) During a home visit, the nurse evaluates teaching provide to a client with type 1 diabetes
mellitus on the ability to prepare an insulin injection. List the order in which the client prepared
the injection that indicates teaching has been effective.
1. Wipe the selected skin site with alcohol and wait for it to dry.
2. Fill the syringe with an amount of air equal to the number of units of insulin, and insert the
needle into the vial.
3. Insert the insulin.
4. Push air into the vial, invert the vial, and withdraw the prescribed units of insulin.
5. Pinch up a fold of skin, and insert the needle into the tissue.
6. Withdraw the needle and apply firm pressure to the site for a few seconds.
Answer: 2, 4, 1, 5, 3, 6
Explanation: The correct order, which indicates effective client teaching, is: fill the syringe with
an amount of air equal to the number of units of insulin, and insert the needle into the vial; push
air into the vial, invert the vial, and withdraw the prescribed units of insulin; wipe the selected
skin site with alcohol and wait for it to dry; pinch up a fold of skin, and insert the needle into the
tissue; insert the insulin; withdraw the needle and apply firm pressure to the site for a few
seconds.
Page Ref: 755
Cognitive Level: Analyzing
Client Need: Physiological Integrity
Nursing Process: Evaluation
Learning Outcome: 7. Evaluate expected outcomes for an individual with diabetes mellitus.

14
Copyright © 2015 Pearson Education, Inc.
7) The healthcare provider prescribes sitagliptin (Januvia) for a client with type 2 diabetes
mellitus. For which potential side effect should the nurse monitor in this client?
A) Elevated blood lipid levels
B) Hyperglycemia
C) Pancreatitis
D) Renal insufficiency
Answer: C
Explanation: A) A potential side effect of sitagliptin (Januvia) is pancreatitis, and the client must
be monitored for this. Sitagliptin (Januvia) does not cause elevated blood lipids, hyperglycemia,
or renal insufficiency.
B) A potential side effect of sitagliptin (Januvia) is pancreatitis, and the client must be monitored
for this. Sitagliptin (Januvia) does not cause elevated blood lipids, hyperglycemia, or renal
insufficiency.
C) A potential side effect of sitagliptin (Januvia) is pancreatitis, and the client must be monitored
for this. Sitagliptin (Januvia) does not cause elevated blood lipids, hyperglycemia, or renal
insufficiency.
D) A potential side effect of sitagliptin (Januvia) is pancreatitis, and the client must be monitored
for this. Sitagliptin (Januvia) does not cause elevated blood lipids, hyperglycemia, or renal
insufficiency.
Page Ref: 756
Cognitive Level: Applying
Client Need: Physiological Integrity
Nursing Process: Implementation
Learning Outcome: 5. Summarize therapies used by interdisciplinary teams in the collaborative
care of an individual with diabetes mellitus.

15
Copyright © 2015 Pearson Education, Inc.
8) A client newly diagnosed with type 1 diabetes mellitus tells the nurse that the diagnosis must
be wrong because the client is not overweight, eats all of the time, and is thin. What should the
nurse respond to the client?
A) "Thin people can be diabetic, too."
B) "Your condition makes it impossible for you to gain weight."
C) "Your lab tests indicate the presence of diabetes."
D) "You are eating large quantities because your condition makes it difficult for your body to
obtain energy from the foods taken in."
Answer: D
Explanation: A) The diabetic client is unable to obtain the needed glucose for the body's cells,
due to the lack of insulin. Patients diagnosed with type 1 diabetes mellitus experience
polyphagia, and are often thin. While the statement about diabetics being thin is correct, it does
not answer the client. It is not impossible for diabetics to gain weight. Although the laboratory
tests might indicate the presence of diabetes, this does not meet the client's needs for teaching.
B) The diabetic client is unable to obtain the needed glucose for the body's cells, due to the lack
of insulin. Patients diagnosed with type 1 diabetes mellitus experience polyphagia, and are often
thin. While the statement about diabetics being thin is correct, it does not answer the client. It is
not impossible for diabetics to gain weight. Although the laboratory tests might indicate the
presence of diabetes, this does not meet the client's needs for teaching.
C) The diabetic client is unable to obtain the needed glucose for the body's cells, due to the lack
of insulin. Patients diagnosed with type 1 diabetes mellitus experience polyphagia, and are often
thin. While the statement about diabetics being thin is correct, it does not answer the client. It is
not impossible for diabetics to gain weight. Although the laboratory tests might indicate the
presence of diabetes, this does not meet the client's needs for teaching.
D) The diabetic client is unable to obtain the needed glucose for the body's cells, due to the lack
of insulin. Patients diagnosed with type 1 diabetes mellitus experience polyphagia, and are often
thin. While the statement about diabetics being thin is correct, it does not answer the client. It is
not impossible for diabetics to gain weight. Although the laboratory tests might indicate the
presence of diabetes, this does not meet the client's needs for teaching.
Page Ref: 740
Cognitive Level: Applying
Client Need: Physiological Integrity
Nursing Process: Implementation
Learning Outcome: 1. Describe the pathophysiology, etiology, clinical manifestations, and
direct and indirect causes of diabetes mellitus.

16
Copyright © 2015 Pearson Education, Inc.
Exemplar 12.2 Diabetes in Children

1) The nurse is concerned that a school-age child has undiagnosed type 1 diabetes mellitus and is
experiencing diabetic ketoacidosis. What did the nurse assess in the client to come to this
conclusion?
Select all that apply.
A) Blurred vision
B) Irregular heartbeat
C) Sunken eye sockets
D) Sluggish bowel sounds
E) Dry mucous membranes
Answer: A, B, C, E
Explanation: A) The clinical manifestations of all forms of diabetes in children include blurred
vision. The clinical hallmarks of diabetic ketoacidosis are dehydration and electrolyte imbalance.
An irregular heartbeat can occur with an electrolyte imbalance. Sunken eye sockets and dry
mucous membranes are seen in dehydration. Sluggish bowel sounds are not an indication of
diabetes or diabetic ketoacidosis.
B) The clinical manifestations of all forms of diabetes in children include blurred vision. The
clinical hallmarks of diabetic ketoacidosis are dehydration and electrolyte imbalance. An
irregular heartbeat can occur with an electrolyte imbalance. Sunken eye sockets and dry mucous
membranes are seen in dehydration. Sluggish bowel sounds are not an indication of diabetes or
diabetic ketoacidosis.
C) The clinical manifestations of all forms of diabetes in children include blurred vision. The
clinical hallmarks of diabetic ketoacidosis are dehydration and electrolyte imbalance. An
irregular heartbeat can occur with an electrolyte imbalance. Sunken eye sockets and dry mucous
membranes are seen in dehydration. Sluggish bowel sounds are not an indication of diabetes or
diabetic ketoacidosis.
D) The clinical manifestations of all forms of diabetes in children include blurred vision. The
clinical hallmarks of diabetic ketoacidosis are dehydration and electrolyte imbalance. An
irregular heartbeat can occur with an electrolyte imbalance. Sunken eye sockets and dry mucous
membranes are seen in dehydration. Sluggish bowel sounds are not an indication of diabetes or
diabetic ketoacidosis.
E) The clinical manifestations of all forms of diabetes in children include blurred vision. The
clinical hallmarks of diabetic ketoacidosis are dehydration and electrolyte imbalance. An
irregular heartbeat can occur with an electrolyte imbalance. Sunken eye sockets and dry mucous
membranes are seen in dehydration. Sluggish bowel sounds are not an indication of diabetes or
diabetic ketoacidosis.
Page Ref: 767
Cognitive Level: Analyzing
Client Need: Physiological Integrity
Nursing Process: Assessment
Learning Outcome: 1. Describe the pathophysiology, etiology, clinical manifestations, and
direct and indirect causes of diabetes in children.

17
Copyright © 2015 Pearson Education, Inc.
2) After reviewing the population demographics for an urban community, the community health
nurse determines that community members would benefit from teaching on types 1 and 2
diabetes mellitus in children. What information caused the nurse to come to this conclusion?
Select all that apply.
A) 60% of community families have both parents diagnosed with type 2 diabetes mellitus.
B) 35% of school-age children do not routinely receive the annual flu vaccination.
C) 50% of children between the ages of 10 and 19 are African-American.
D) 25% of children between the ages of 10 and 19 are Hispanic.
E) 75% of school-age children are raised in families where both parents are unemployed.
Answer: A, C, D
Explanation: A) The risk factors for diabetes include race, ethnicity, and family history. Type 2
diabetes mellitus rates are greater among youth ages 10-19 with higher rates among U.S.
minority populations than in non-Hispanic Whites. Frequency of obtaining the annual flu
vaccination and socioeconomic status are not risk factors for the development of type 1 or 2
diabetes in children.
B) The risk factors for diabetes include race, ethnicity, and family history. Type 2 diabetes
mellitus rates are greater among youth ages 10-19 with higher rates among U.S. minority
populations than in non-Hispanic Whites. Frequency of obtaining the annual flu vaccination and
socioeconomic status are not risk factors for the development of type 1 or 2 diabetes in children.
C) The risk factors for diabetes include race, ethnicity, and family history. Type 2 diabetes
mellitus rates are greater among youth ages 10-19 with higher rates among U.S. minority
populations than in non-Hispanic Whites. Frequency of obtaining the annual flu vaccination and
socioeconomic status are not risk factors for the development of type 1 or 2 diabetes in children.
D) The risk factors for diabetes include race, ethnicity, and family history. Type 2 diabetes
mellitus rates are greater among youth ages 10-19 with higher rates among U.S. minority
populations than in non-Hispanic Whites. Frequency of obtaining the annual flu vaccination and
socioeconomic status are not risk factors for the development of type 1 or 2 diabetes in children.
E) The risk factors for diabetes include race, ethnicity, and family history. Type 2 diabetes
mellitus rates are greater among youth ages 10-19 with higher rates among U.S. minority
populations than in non-Hispanic Whites. Frequency of obtaining the annual flu vaccination and
socioeconomic status are not risk factors for the development of type 1 or 2 diabetes in children.
Page Ref: 767
Cognitive Level: Analyzing
Client Need: Health Promotion and Maintenance
Nursing Process: Assessment
Learning Outcome: 2. Identify risk factors and prevention methods associated with diabetes in
children.

18
Copyright © 2015 Pearson Education, Inc.
3) The Intensive Care nurse is preparing to admit a school-age child for treatment of diabetic
ketoacidosis. On what should the nurse focus for this client's care?
Select all that apply.
A) Peripheral perfusion
B) Fluid volume overload
C) Frequent blood glucose monitoring
D) Intravenous fluid infusions
E) Insulin infusion
Answer: B, C, D, E
Explanation: A) For the child experiencing diabetic ketoacidosis, frequent blood sugar
monitoring, IV fluids, and insulin drips for treatment mandate that the child be cared for in an
Intensive Care environment until stabilized. The child will be dehydrated and most likely will
not need treatment for fluid volume overload. The child has not lost any blood volume, so
peripheral perfusion will most likely not be a concern.
B) For the child experiencing diabetic ketoacidosis, frequent blood sugar monitoring, IV fluids,
and insulin drips for treatment mandate that the child be cared for in an Intensive Care
environment until stabilized. The child will be dehydrated and most likely will not need
treatment for fluid volume overload. The child has not lost any blood volume, so peripheral
perfusion will most likely not be a concern.
C) For the child experiencing diabetic ketoacidosis, frequent blood sugar monitoring, IV fluids,
and insulin drips for treatment mandate that the child be cared for in an Intensive Care
environment until stabilized. The child will be dehydrated and most likely will not need
treatment for fluid volume overload. The child has not lost any blood volume, so peripheral
perfusion will most likely not be a concern.
D) For the child experiencing diabetic ketoacidosis, frequent blood sugar monitoring, IV fluids,
and insulin drips for treatment mandate that the child be cared for in an Intensive Care
environment until stabilized. The child will be dehydrated and most likely will not need
treatment for fluid volume overload. The child has not lost any blood volume, so peripheral
perfusion will most likely not be a concern.
E) For the child experiencing diabetic ketoacidosis, frequent blood sugar monitoring, IV fluids,
and insulin drips for treatment mandate that the child be cared for in an Intensive Care
environment until stabilized. The child will be dehydrated and most likely will not need
treatment for fluid volume overload. The child has not lost any blood volume, so peripheral
perfusion will most likely not be a concern.
Page Ref: 767
Cognitive Level: Applying
Client Need: Physiological Integrity
Nursing Process: Planning
Learning Outcome: 3. Illustrate the nursing process in providing culturally competent care for
children with diabetes.

19
Copyright © 2015 Pearson Education, Inc.
4) The nurse is planning care for a 6-year-old child newly diagnosed with type 1 diabetes
mellitus. The child's mother appears unconcerned with the diagnosis and is complaining about
the cost of medication, as three additional children in the family have needs. On which nursing
diagnoses should the nurse focus when planning this client's care?
Select all that apply.
A) Chronic Pain
B) Knowledge Deficit
C) Ineffective Coping (Family)
D) Risk for Unstable Blood Glucose
E) Risk for Injury
Answer: B, C, D
Explanation: A) The child is only 6, so the mother will need to learn how to provide the insulin
injections; however, the mother is unconcerned with the diagnosis. This is a Knowledge Deficit.
The mother's complaint about the cost of medication would cause the client to have a Risk for
Unstable Blood Glucose, as it is unknown whether the mother is going to ensure the child has the
required medication. The family has a total of four children and the mother is concerned with
money for medication, as there are three additional children with needs. This could lead to
Ineffective Family Coping. There is no evidence to support the diagnoses of Chronic Pain or
Risk for Injury with this client.
B) The child is only 6, so the mother will need to learn how to provide the insulin injections;
however, the mother is unconcerned with the diagnosis. This is a Knowledge Deficit. The
mother's complaint about the cost of medication would cause the client to have a Risk for
Unstable Blood Glucose, as it is unknown whether the mother is going to ensure the child has the
required medication. The family has a total of four children and the mother is concerned with
money for medication, as there are three additional children with needs. This could lead to
Ineffective Family Coping. There is no evidence to support the diagnoses of Chronic Pain or
Risk for Injury with this client.
C) The child is only 6, so the mother will need to learn how to provide the insulin injections;
however, the mother is unconcerned with the diagnosis. This is a Knowledge Deficit. The
mother's complaint about the cost of medication would cause the client to have a Risk for
Unstable Blood Glucose, as it is unknown whether the mother is going to ensure the child has the
required medication. The family has a total of four children and the mother is concerned with
money for medication, as there are three additional children with needs. This could lead to
Ineffective Family Coping. There is no evidence to support the diagnoses of Chronic Pain or
Risk for Injury with this client.
D) The child is only 6, so the mother will need to learn how to provide the insulin injections;
however, the mother is unconcerned with the diagnosis. This is a Knowledge Deficit. The
mother's complaint about the cost of medication would cause the client to have a Risk for
Unstable Blood Glucose, as it is unknown whether the mother is going to ensure the child has the
required medication. The family has a total of four children and the mother is concerned with
money for medication, as there are three additional children with needs. This could lead to
Ineffective Family Coping. There is no evidence to support the diagnoses of Chronic Pain or
Risk for Injury with this client.

20
Copyright © 2015 Pearson Education, Inc.
E) The child is only 6, so the mother will need to learn how to provide the insulin injections;
however, the mother is unconcerned with the diagnosis. This is a Knowledge Deficit. The
mother's complaint about the cost of medication would cause the client to have a Risk for
Unstable Blood Glucose, as it is unknown whether the mother is going to ensure the child has the
required medication. The family has a total of four children and the mother is concerned with
money for medication, as there are three additional children with needs. This could lead to
Ineffective Family Coping. There is no evidence to support the diagnoses of Chronic Pain or
Risk for Injury with this client.
Page Ref: 775
Cognitive Level: Analyzing
Client Need: Physiological Integrity
Nursing Process: Planning
Learning Outcome: 4. Formulate priority nursing diagnoses appropriate for a child with
diabetes.

21
Copyright © 2015 Pearson Education, Inc.
5) A multidisciplinary conference has concluded that focused on the care needs of a 10-year-old
child with type 2 diabetes mellitus. On which areas should the team focus care to improve this
client's long-term prognosis?
Select all that apply.
A) Weaning off oral medications
B) Food intake based on age, sex, and physical activity
C) Obtaining adequate rest and sleep
D) Physical activity to be at least 30-60 minutes per day most days of the week
E) Family participation in the lifestyle change
Answer: B, D, E
Explanation: A) The child with type 2 diabetes mellitus will most likely be treated with oral
hypoglycemic agents. Weaning off of these medications will not improve the client's long-term
prognosis. The child with type 2 diabetes mellitus does not have a need for adequate rest and
sleep to improve the long-term prognosis. Plans to improve the client's long-term prognosis
should focus on food intake that is based on the client's age, sex, and physical activity, obtaining
the required physical activity that is recommended for most days of the week, and family support
to comply with lifestyle changes that the client needs.
B) The child with type 2 diabetes mellitus will most likely be treated with oral hypoglycemic
agents. Weaning off of these medications will not improve the client's long-term prognosis. The
child with type 2 diabetes mellitus does not have a need for adequate rest and sleep to improve
the long-term prognosis. Plans to improve the client's long-term prognosis should focus on food
intake that is based on the client's age, sex, and physical activity, obtaining the required physical
activity that is recommended for most days of the week, and family support to comply with
lifestyle changes that the client needs.
C) The child with type 2 diabetes mellitus will most likely be treated with oral hypoglycemic
agents. Weaning off of these medications will not improve the client's long-term prognosis. The
child with type 2 diabetes mellitus does not have a need for adequate rest and sleep to improve
the long-term prognosis. Plans to improve the client's long-term prognosis should focus on food
intake that is based on the client's age, sex, and physical activity, obtaining the required physical
activity that is recommended for most days of the week, and family support to comply with
lifestyle changes that the client needs.
D) The child with type 2 diabetes mellitus will most likely be treated with oral hypoglycemic
agents. Weaning off of these medications will not improve the client's long-term prognosis. The
child with type 2 diabetes mellitus does not have a need for adequate rest and sleep to improve
the long-term prognosis. Plans to improve the client's long-term prognosis should focus on food
intake that is based on the client's age, sex, and physical activity, obtaining the required physical
activity that is recommended for most days of the week, and family support to comply with
lifestyle changes that the client needs.

22
Copyright © 2015 Pearson Education, Inc.
E) The child with type 2 diabetes mellitus will most likely be treated with oral hypoglycemic
agents. Weaning off of these medications will not improve the client's long-term prognosis. The
child with type 2 diabetes mellitus does not have a need for adequate rest and sleep to improve
the long-term prognosis. Plans to improve the client's long-term prognosis should focus on food
intake that is based on the client's age, sex, and physical activity, obtaining the required physical
activity that is recommended for most days of the week, and family support to comply with
lifestyle changes that the client needs.
Page Ref: 776-777
Cognitive Level: Creating
Client Need: Health Promotion and Maintenance
Nursing Process: Planning
Learning Outcome: 5. Summarize therapies used by interdisciplinary teams in the collaborative
care of a child with diabetes.

23
Copyright © 2015 Pearson Education, Inc.
6) The nurse is finalizing a plan of care for a school-age child newly diagnosed with type 1
diabetes mellitus. On which areas should the plan focus to achieve the maximum outcomes for
this client?
Select all that apply.
A) Ways to minimize the number of school days missed
B) Identification and referral to community resources
C) Physical activities that limit exposure to injuries
D) Self-management of glucose monitoring and medications
E) Signs and symptoms of hypoglycemia and actions to take
Answer: B, D, E
Explanation: A) Planning for care of children with diabetes depends on the assessment findings.
Planning should begin at the time of diagnosis and move forward to prepare the child and the
family for self-management of glucose monitoring and medications, signs and symptoms of
hypoglycemia, and actions to take. Before discharge, the child and the family should be linked to
the resources in the community that will support care of the child with diabetes. Minimizing the
number of school days missed and activities that limit exposure to injuries are not immediate
priorities for this client's plan.
B) Planning for care of children with diabetes depends on the assessment findings. Planning
should begin at the time of diagnosis and move forward to prepare the child and the family for
self-management of glucose monitoring and medications, signs and symptoms of hypoglycemia,
and actions to take. Before discharge, the child and the family should be linked to the resources
in the community that will support care of the child with diabetes. Minimizing the number of
school days missed and activities that limit exposure to injuries are not immediate priorities for
this client's plan.
C) Planning for care of children with diabetes depends on the assessment findings. Planning
should begin at the time of diagnosis and move forward to prepare the child and the family for
self-management of glucose monitoring and medications, signs and symptoms of hypoglycemia,
and actions to take. Before discharge, the child and the family should be linked to the resources
in the community that will support care of the child with diabetes. Minimizing the number of
school days missed and activities that limit exposure to injuries are not immediate priorities for
this client's plan.
D) Planning for care of children with diabetes depends on the assessment findings. Planning
should begin at the time of diagnosis and move forward to prepare the child and the family for
self-management of glucose monitoring and medications, signs and symptoms of hypoglycemia,
and actions to take. Before discharge, the child and the family should be linked to the resources
in the community that will support care of the child with diabetes. Minimizing the number of
school days missed and activities that limit exposure to injuries are not immediate priorities for
this client's plan.

24
Copyright © 2015 Pearson Education, Inc.
E) Planning for care of children with diabetes depends on the assessment findings. Planning
should begin at the time of diagnosis and move forward to prepare the child and the family for
self-management of glucose monitoring and medications, signs and symptoms of hypoglycemia,
and actions to take. Before discharge, the child and the family should be linked to the resources
in the community that will support care of the child with diabetes. Minimizing the number of
school days missed and activities that limit exposure to injuries are not immediate priorities for
this client's plan.
Page Ref: 778
Cognitive Level: Creating
Client Need: Health Promotion and Maintenance
Nursing Process: Planning
Learning Outcome: 6. Plan evidence-based care for a child with diabetes and his or her family in
collaboration with other members of the healthcare team.

25
Copyright © 2015 Pearson Education, Inc.
7) The nurse is evaluating care provided to a school-age child recently discharged from the
hospital for type 1 diabetes mellitus care. Which observations indicate that care outcomes have
been achieved?
Select all that apply.
A) The client is documenting blood glucose readings and associated insulin dosages in a
notebook next to the glucometer.
B) The client spends attends school and completes homework before bedtime.
C) The client correctly demonstrates drawing up and administering daily insulin dose.
D) The client has a glucagon kit in school backpack and explains how it should be used.
E) The client watches brothers play Little League baseball on the weekends.
Answer: A, C, D
Explanation: A) Evidence that care provided to a school-age child with type 1 diabetes mellitus
has been effective include documentation of blood glucose readings with associated insulin
dosages, demonstrating the correct technique to draw and self-inject daily insulin dosages, and
having an emergency glucagon kit available for use if necessary. Attending school, doing
homework and watching family members participate in sports are not meeting the client's needs
for physical activity and would indicate that care has not been effective.
B) Evidence that care provided to a school-age child with type 1 diabetes mellitus has been
effective include documentation of blood glucose readings with associated insulin dosages,
demonstrating the correct technique to draw and self-inject daily insulin dosages, and having an
emergency glucagon kit available for use if necessary. Attending school, doing homework and
watching family members participate in sports are not meeting the client's needs for physical
activity and would indicate that care has not been effective.
C) Evidence that care provided to a school-age child with type 1 diabetes mellitus has been
effective include documentation of blood glucose readings with associated insulin dosages,
demonstrating the correct technique to draw and self-inject daily insulin dosages, and having an
emergency glucagon kit available for use if necessary. Attending school, doing homework and
watching family members participate in sports are not meeting the client's needs for physical
activity and would indicate that care has not been effective.
D) Evidence that care provided to a school-age child with type 1 diabetes mellitus has been
effective include documentation of blood glucose readings with associated insulin dosages,
demonstrating the correct technique to draw and self-inject daily insulin dosages, and having an
emergency glucagon kit available for use if necessary. Attending school, doing homework and
watching family members participate in sports are not meeting the client's needs for physical
activity and would indicate that care has not been effective.
E) Evidence that care provided to a school-age child with type 1 diabetes mellitus has been
effective include documentation of blood glucose readings with associated insulin dosages,
demonstrating the correct technique to draw and self-inject daily insulin dosages, and having an
emergency glucagon kit available for use if necessary. Attending school, doing homework and
watching family members participate in sports are not meeting the client's needs for physical
activity and would indicate that care has not been effective.
Page Ref: 778
Cognitive Level: Analyzing
Client Need: Health Promotion and Maintenance
Nursing Process: Evaluation
Learning Outcome: 7. Evaluate expected outcomes for a child with diabetes.

26
Copyright © 2015 Pearson Education, Inc.
Exemplar 12.3 Liver Disease

1) The nurse is concerned that a client with potential hepatic failure is at risk for developing
ascites. Which assessment finding would indicate this development?
A) Increased abdominal girth
B) Gallbladder pain
C) Yellow-tinged skin
D) Bleeding and bruising easily
Answer: A
Explanation: A) Ascites is the accumulation of the fluid in the abdomen, and is a result of liver
failure. The client with ascites would have an increased abdominal girth. Jaundice is manifested
as yellow-tinged skin, and is the result of hepatic disorders. The client experiencing hepatic
problems might have bleeding and bruising issues due to inadequate vitamin K. Obstructed
biliary flow could be the cause of gallbladder pain.
B) Ascites is the accumulation of the fluid in the abdomen, and is a result of liver failure. The
client with ascites would have an increased abdominal girth. Jaundice is manifested as yellow-
tinged skin, and is the result of hepatic disorders. The client experiencing hepatic problems might
have bleeding and bruising issues due to inadequate vitamin K. Obstructed biliary flow could be
the cause of gallbladder pain.
C) Ascites is the accumulation of the fluid in the abdomen, and is a result of liver failure. The
client with ascites would have an increased abdominal girth. Jaundice is manifested as yellow-
tinged skin, and is the result of hepatic disorders. The client experiencing hepatic problems might
have bleeding and bruising issues due to inadequate vitamin K. Obstructed biliary flow could be
the cause of gallbladder pain.
D) Ascites is the accumulation of the fluid in the abdomen, and is a result of liver failure. The
client with ascites would have an increased abdominal girth. Jaundice is manifested as yellow-
tinged skin, and is the result of hepatic disorders. The client experiencing hepatic problems might
have bleeding and bruising issues due to inadequate vitamin K. Obstructed biliary flow could be
the cause of gallbladder pain.
Page Ref: 782
Cognitive Level: Analyzing
Client Need: Physiological Integrity
Nursing Process: Assessment
Learning Outcome: 1. Describe the pathophysiology, etiology, clinical manifestations, and
direct and indirect causes of liver disease.

27
Copyright © 2015 Pearson Education, Inc.
2) The nurse is caring for a client with cirrhosis of the liver. Which information in the client's
health history supports this diagnosis?
A) Smokes two packs of cigarettes per day.
B) Drinks a six-pack of beer each evening.
C) Eats salads for lunch every day.
D) Plays on an adult softball team several times a week.
Answer: B
Explanation: A) Risk factors for the development of cirrhosis of the liver include excessive
alcohol intake. Smoking, ingestion of salads, and exercise are not risk factors for the
development of this health problem.
B) Risk factors for the development of cirrhosis of the liver include excessive alcohol intake.
Smoking, ingestion of salads, and exercise are not risk factors for the development of this health
problem.
C) Risk factors for the development of cirrhosis of the liver include excessive alcohol intake.
Smoking, ingestion of salads, and exercise are not risk factors for the development of this health
problem.
D) Risk factors for the development of cirrhosis of the liver include excessive alcohol intake.
Smoking, ingestion of salads, and exercise are not risk factors for the development of this health
problem.
Page Ref: 782
Cognitive Level: Analyzing
Client Need: Physiological Integrity
Nursing Process: Assessment
Learning Outcome: 2. Identify risk factors and prevention methods associated with liver disease.

28
Copyright © 2015 Pearson Education, Inc.
3) The community health nurse is planning education for a group of individuals from Alcoholics
Anonymous on the risk factors for liver disease. The group has a high number of Native
Americans in attendance. What should the nurse explain as the reasons for the high incidence of
cirrhosis in this ethnic group?
Select all that apply.
A) Pollution
B) Variations in alcohol metabolism
C) Stress due to socioeconomic factors
D) Consuming alcohol with food
E) Climate
Answer: B, C
Explanation: A) Alcohol consumption is the sixth-leading cause of death for Native Americans,
particularly Alaskans. It is thought that contributing factors include variations in alcohol
metabolism, socioeconomic factors that lead to stress, and, consuming alcohol without food.
Climate and pollution are not factors.
B) Alcohol consumption is the sixth-leading cause of death for Native Americans, particularly
Alaskans. It is thought that contributing factors include variations in alcohol metabolism,
socioeconomic factors that lead to stress, and, consuming alcohol without food. Climate and
pollution are not factors.
C) Alcohol consumption is the sixth-leading cause of death for Native Americans, particularly
Alaskans. It is thought that contributing factors include variations in alcohol metabolism,
socioeconomic factors that lead to stress, and, consuming alcohol without food. Climate and
pollution are not factors.
D) Alcohol consumption is the sixth-leading cause of death for Native Americans, particularly
Alaskans. It is thought that contributing factors include variations in alcohol metabolism,
socioeconomic factors that lead to stress, and, consuming alcohol without food. Climate and
pollution are not factors.
E) Alcohol consumption is the sixth-leading cause of death for Native Americans, particularly
Alaskans. It is thought that contributing factors include variations in alcohol metabolism,
socioeconomic factors that lead to stress, and, consuming alcohol without food. Climate and
pollution are not factors.
Page Ref: 782
Cognitive Level: Applying
Client Need: Health Promotion and Maintenance
Nursing Process: Implementation
Learning Outcome: 3. Illustrate the nursing process in providing culturally competent care
across the life span for individuals with liver disease.

29
Copyright © 2015 Pearson Education, Inc.
4) A nurse is caring for a client who was recently admitted for treatment of cirrhosis. The client
is currently experiencing BP of 200/100, +3 pitting edema, and shortness of breath. Which
diagnosis should the nurse select as a priority for this client?
A) Excess Fluid Volume
B) Ineffective Tissue Perfusion
C) Deficient Fluid Volume
D) Impaired Skin Integrity
Answer: A
Explanation: A) The client experiencing shortness of breath, edema, and hypertension should
have a care plan for fluid volume excess. Hypertension, shortness of breath, and edema are
manifestations of fluid excess. Hypotension and dry mucous membranes are associated with
deficient fluid volume. Ineffective Tissue Perfusion would be the appropriate diagnosis for a
client experiencing cyanosis or tissue necrosis. Edema can cause an alteration in skin integrity,
but there is no evidence of such problems with this client.
B) The client experiencing shortness of breath, edema, and hypertension should have a care plan
for fluid volume excess. Hypertension, shortness of breath, and edema are manifestations of fluid
excess. Hypotension and dry mucous membranes are associated with deficient fluid volume.
Ineffective Tissue Perfusion would be the appropriate diagnosis for a client experiencing
cyanosis or tissue necrosis. Edema can cause an alteration in skin integrity, but there is no
evidence of such problems with this client.
C) The client experiencing shortness of breath, edema, and hypertension should have a care plan
for fluid volume excess. Hypertension, shortness of breath, and edema are manifestations of fluid
excess. Hypotension and dry mucous membranes are associated with deficient fluid volume.
Ineffective Tissue Perfusion would be the appropriate diagnosis for a client experiencing
cyanosis or tissue necrosis. Edema can cause an alteration in skin integrity, but there is no
evidence of such problems with this client.
D) The client experiencing shortness of breath, edema, and hypertension should have a care plan
for fluid volume excess. Hypertension, shortness of breath, and edema are manifestations of fluid
excess. Hypotension and dry mucous membranes are associated with deficient fluid volume.
Ineffective Tissue Perfusion would be the appropriate diagnosis for a client experiencing
cyanosis or tissue necrosis. Edema can cause an alteration in skin integrity, but there is no
evidence of such problems with this client.
Page Ref: 786
Cognitive Level: Analyzing
Client Need: Health Promotion and Maintenance
Nursing Process: Planning
Learning Outcome: 4. Formulate priority nursing diagnoses appropriate for an individual with
liver disease.

30
Copyright © 2015 Pearson Education, Inc.
5) The family of a client with cirrhosis of the liver asks what symptoms they need to look for
while the client is being cared for in their home. What should the nurse teach the family that
indicates portal hypertension in this client?
A) Muscle wasting
B) Hypothermia
C) Bleeding gums
D) Hemorrhoids
Answer: D
Explanation: A) Obstruction to portal blood flow causes a rise in portal venous pressure,
resulting in splenomegaly, ascites, and dilation of collateral venous channels predominately in
the paraumbilical and hemorrhoidal veins and the cardia of the stomach, and extending into the
esophagus. Bleeding gums indicate insufficient vitamin K production in the liver. Muscle
wasting is commonly associated with the poor nutritional intake seen in clients with cirrhosis.
Hypothermia is an unrelated finding.
B) Obstruction to portal blood flow causes a rise in portal venous pressure, resulting in
splenomegaly, ascites, and dilation of collateral venous channels predominately in the
paraumbilical and hemorrhoidal veins and the cardia of the stomach, and extending into the
esophagus. Bleeding gums indicate insufficient vitamin K production in the liver. Muscle
wasting is commonly associated with the poor nutritional intake seen in clients with cirrhosis.
Hypothermia is an unrelated finding.
C) Obstruction to portal blood flow causes a rise in portal venous pressure, resulting in
splenomegaly, ascites, and dilation of collateral venous channels predominately in the
paraumbilical and hemorrhoidal veins and the cardia of the stomach, and extending into the
esophagus. Bleeding gums indicate insufficient vitamin K production in the liver. Muscle
wasting is commonly associated with the poor nutritional intake seen in clients with cirrhosis.
Hypothermia is an unrelated finding.
D) Obstruction to portal blood flow causes a rise in portal venous pressure, resulting in
splenomegaly, ascites, and dilation of collateral venous channels predominately in the
paraumbilical and hemorrhoidal veins and the cardia of the stomach, and extending into the
esophagus. Bleeding gums indicate insufficient vitamin K production in the liver. Muscle
wasting is commonly associated with the poor nutritional intake seen in clients with cirrhosis.
Hypothermia is an unrelated finding.
Page Ref: 786
Cognitive Level: Applying
Client Need: Physiological Integrity
Nursing Process: Assessment
Learning Outcome: 6. Plan evidence-based care for an individual with liver disease and his or
her family in collaboration with other members of the healthcare team.

31
Copyright © 2015 Pearson Education, Inc.
6) The nurse is caring for client recovering from a liver transplant necessitated by cirrhosis of the
liver. Which postoperative outcome would be a priority for this client?
A) Moist membranes of the mouth
B) Normal serum bilirubin levels
C) Ability to move the legs
D) Normal pupil reaction
Answer: B
Explanation: A) Normal bilirubin levels would indicate that the transplanted liver is functioning
correctly. Normal pupil reaction, leg movement, and moist mouth membranes are all normal
findings for any postoperative client.
B) Normal bilirubin levels would indicate that the transplanted liver is functioning correctly.
Normal pupil reaction, leg movement, and moist mouth membranes are all normal findings for
any postoperative client.
C) Normal bilirubin levels would indicate that the transplanted liver is functioning correctly.
Normal pupil reaction, leg movement, and moist mouth membranes are all normal findings for
any postoperative client.
D) Normal bilirubin levels would indicate that the transplanted liver is functioning correctly.
Normal pupil reaction, leg movement, and moist mouth membranes are all normal findings for
any postoperative client.
Page Ref: 786
Cognitive Level: Analyzing
Client Need: Physiological Integrity
Nursing Process: Evaluation
Learning Outcome: 7. Evaluate expected outcomes for an individual with liver disease.

32
Copyright © 2015 Pearson Education, Inc.
7) A client with liver cirrhosis begins to drain bright red blood through the nasogastric tube.
What should the nurse prepare to administer to this client?
A) Vitamin K
B) Ferrous sulfate
C) Platelets
D) Folic acid
Answer: C
Explanation: A) Ferrous sulfate and folic acid are given as indicated to treat anemia. Vitamin K
may be ordered to reduce the risk of bleeding. When bleeding is acute, packed RBCs, fresh
frozen plasma, or platelets may be administered to restore blood components and promote
hemostasis.
B) Ferrous sulfate and folic acid are given as indicated to treat anemia. Vitamin K may be
ordered to reduce the risk of bleeding. When bleeding is acute, packed RBCs, fresh frozen
plasma, or platelets may be administered to restore blood components and promote hemostasis.
C) Ferrous sulfate and folic acid are given as indicated to treat anemia. Vitamin K may be
ordered to reduce the risk of bleeding. When bleeding is acute, packed RBCs, fresh frozen
plasma, or platelets may be administered to restore blood components and promote hemostasis.
D) Ferrous sulfate and folic acid are given as indicated to treat anemia. Vitamin K may be
ordered to reduce the risk of bleeding. When bleeding is acute, packed RBCs, fresh frozen
plasma, or platelets may be administered to restore blood components and promote hemostasis.
Page Ref: 785
Cognitive Level: Applying
Client Need: Physiological Integrity
Nursing Process: Implementation
Learning Outcome: 5. Summarize therapies used by interdisciplinary teams in the collaborative
care of an individual with liver disease.

33
Copyright © 2015 Pearson Education, Inc.
8) The nurse determines that a client is at risk for contracting hepatitis B because of intravenous
drug use. What should the nurse teach to reduce the client's risk for this health problem?
A) Avoid contaminated food and water.
B) Avoid sharing needles.
C) Avoid alcohol consumption.
D) Wash hands frequently, as the disease is transmitted via the fecal-oral route.
Answer: B
Explanation: A) Hepatitis B is contracted through contaminated blood and body fluids. The
client will increase the risk of contracting hepatitis B by sharing needles. Hepatitis A is
transmitted via the fecal-oral route. Laënnec's cirrhosis is the result of alcohol and hepatitis B
and C. Contaminated food and water causes hepatitis A, not B.
B) Hepatitis B is contracted through contaminated blood and body fluids. The client will increase
the risk of contracting hepatitis B by sharing needles. Hepatitis A is transmitted via the fecal-oral
route. Laënnec's cirrhosis is the result of alcohol and hepatitis B and C. Contaminated food and
water causes hepatitis A, not B.
C) Hepatitis B is contracted through contaminated blood and body fluids. The client will increase
the risk of contracting hepatitis B by sharing needles. Hepatitis A is transmitted via the fecal-oral
route. Laënnec's cirrhosis is the result of alcohol and hepatitis B and C. Contaminated food and
water causes hepatitis A, not B.
D) Hepatitis B is contracted through contaminated blood and body fluids. The client will
increase the risk of contracting hepatitis B by sharing needles. Hepatitis A is transmitted via the
fecal-oral route. Laënnec's cirrhosis is the result of alcohol and hepatitis B and C. Contaminated
food and water causes hepatitis A, not B.
Page Ref: 782
Cognitive Level: Applying
Client Need: Health Promotion and Maintenance
Nursing Process: Implementation
Learning Outcome: 2. Identify risk factors and prevention methods associated with liver disease.

34
Copyright © 2015 Pearson Education, Inc.
9) A nurse is caring for a client with end-stage liver disease. Which hematological alterations
might the nurse anticipate with this client?
Select all that apply.
A) Elevated serum albumin levels due to increased protein synthesis
B) Decreased clotting factor levels due to impaired clotting mechanisms
C) Hyperglycemia due to disrupted glucose metabolism
D) Increased serum vitamin K due to impaired clearance of fat-soluble vitamins
E) Increased plasma oncotic pressure due to impaired protein metabolism
Answer: B, C
Explanation: A) Impaired function of liver cells has multiple effects. Impaired protein
metabolism with decreased production of albumin and clotting factors occurs. Low albumin
levels contribute to edema in peripheral tissues and ascites (accumulation of fluid in the
abdomen), as plasma oncotic pressure is reduced, not increased. Impaired clotting-factor
production increases the risk for bleeding. Disrupted glucose metabolism and storage may result
in hyperglycemia. Also, serum vitamin K is decreased due to impaired absorption of fat-soluble
vitamins.
B) Impaired function of liver cells has multiple effects. Impaired protein metabolism with
decreased production of albumin and clotting factors occurs. Low albumin levels contribute to
edema in peripheral tissues and ascites (accumulation of fluid in the abdomen), as plasma
oncotic pressure is reduced, not increased. Impaired clotting-factor production increases the risk
for bleeding. Disrupted glucose metabolism and storage may result in hyperglycemia. Also,
serum vitamin K is decreased due to impaired absorption of fat-soluble vitamins.
C) Impaired function of liver cells has multiple effects. Impaired protein metabolism with
decreased production of albumin and clotting factors occurs. Low albumin levels contribute to
edema in peripheral tissues and ascites (accumulation of fluid in the abdomen), as plasma
oncotic pressure is reduced, not increased. Impaired clotting-factor production increases the risk
for bleeding. Disrupted glucose metabolism and storage may result in hyperglycemia. Also,
serum vitamin K is decreased due to impaired absorption of fat-soluble vitamins.
D) Impaired function of liver cells has multiple effects. Impaired protein metabolism with
decreased production of albumin and clotting factors occurs. Low albumin levels contribute to
edema in peripheral tissues and ascites (accumulation of fluid in the abdomen), as plasma
oncotic pressure is reduced, not increased. Impaired clotting-factor production increases the risk
for bleeding. Disrupted glucose metabolism and storage may result in hyperglycemia. Also,
serum vitamin K is decreased due to impaired absorption of fat-soluble vitamins.
E) Impaired function of liver cells has multiple effects. Impaired protein metabolism with
decreased production of albumin and clotting factors occurs. Low albumin levels contribute to
edema in peripheral tissues and ascites (accumulation of fluid in the abdomen), as plasma
oncotic pressure is reduced, not increased. Impaired clotting-factor production increases the risk
for bleeding. Disrupted glucose metabolism and storage may result in hyperglycemia. Also,
serum vitamin K is decreased due to impaired absorption of fat-soluble vitamins.
Page Ref: 786
Cognitive Level: Analyzing
Client Need: Physiological Integrity
Nursing Process: Assessment
Learning Outcome: 1. Describe the pathophysiology, etiology, clinical manifestations, and
direct and indirect causes of liver disease.

35
Copyright © 2015 Pearson Education, Inc.
10) A client with liver disease presents to the hospital with severe ascites. The nurse caring for
the client understands that the pathophysiology involved in the development of ascites includes:
Select all that apply.
A) Presence of portal hypertension.
B) Presence of hyperalbuminemia.
C) Increased colloidal osmotic pressure.
D) Sodium and water retention.
E) Presence of hypoaldosteronism.
Answer: A, D
Explanation: A) Ascites is the accumulation of plasma-rich fluid in the abdominal cavity.
Although portal hypertension is the primary cause of ascites, decreased serum proteins and
increased aldosterone also contribute to the fluid accumulation. Hypoalbuminemia (low serum
albumin) decreases the colloidal osmotic pressure of plasma. This pressure normally holds fluid
in the intravascular compartment, but when the plasma colloidal osmotic pressure decreases,
fluid escapes into extravascular compartments. Hyperaldosteronism (an increase in aldosterone)
causes sodium and water retention, contributing to ascites and generalized edema.
B) Ascites is the accumulation of plasma-rich fluid in the abdominal cavity. Although portal
hypertension is the primary cause of ascites, decreased serum proteins and increased aldosterone
also contribute to the fluid accumulation. Hypoalbuminemia (low serum albumin) decreases the
colloidal osmotic pressure of plasma. This pressure normally holds fluid in the intravascular
compartment, but when the plasma colloidal osmotic pressure decreases, fluid escapes into
extravascular compartments. Hyperaldosteronism (an increase in aldosterone) causes sodium and
water retention, contributing to ascites and generalized edema.
C) Ascites is the accumulation of plasma-rich fluid in the abdominal cavity. Although portal
hypertension is the primary cause of ascites, decreased serum proteins and increased aldosterone
also contribute to the fluid accumulation. Hypoalbuminemia (low serum albumin) decreases the
colloidal osmotic pressure of plasma. This pressure normally holds fluid in the intravascular
compartment, but when the plasma colloidal osmotic pressure decreases, fluid escapes into
extravascular compartments. Hyperaldosteronism (an increase in aldosterone) causes sodium and
water retention, contributing to ascites and generalized edema.
D) Ascites is the accumulation of plasma-rich fluid in the abdominal cavity. Although portal
hypertension is the primary cause of ascites, decreased serum proteins and increased aldosterone
also contribute to the fluid accumulation. Hypoalbuminemia (low serum albumin) decreases the
colloidal osmotic pressure of plasma. This pressure normally holds fluid in the intravascular
compartment, but when the plasma colloidal osmotic pressure decreases, fluid escapes into
extravascular compartments. Hyperaldosteronism (an increase in aldosterone) causes sodium and
water retention, contributing to ascites and generalized edema.

36
Copyright © 2015 Pearson Education, Inc.
E) Ascites is the accumulation of plasma-rich fluid in the abdominal cavity. Although portal
hypertension is the primary cause of ascites, decreased serum proteins and increased aldosterone
also contribute to the fluid accumulation. Hypoalbuminemia (low serum albumin) decreases the
colloidal osmotic pressure of plasma. This pressure normally holds fluid in the intravascular
compartment, but when the plasma colloidal osmotic pressure decreases, fluid escapes into
extravascular compartments. Hyperaldosteronism (an increase in aldosterone) causes sodium and
water retention, contributing to ascites and generalized edema.
Page Ref: 782
Cognitive Level: Analyzing
Client Need: Physiological Integrity
Nursing Process: Assessment
Learning Outcome: 1. Describe the pathophysiology, etiology, clinical manifestations, and
direct and indirect causes of liver disease.

Exemplar 12.4 Obesity

1) A nurse working in a community health center is counseling a teen regarding a suspected


eating disorder. The teen is of normal weight but admits to periods of overeating, especially
when experiencing stress. For which potential health problem should the nurse counsel the teen?
A) Obesity
B) An emaciated body
C) Hunger pangs
D) Anorexia
Answer: A
Explanation: A) Individuals may eat to relieve depression or anxiety, which can lead to obesity.
Overeating will not lead to emaciation, hunger pangs, or anorexia.
B) Individuals may eat to relieve depression or anxiety, which can lead to obesity. Overeating
will not lead to emaciation, hunger pangs, or anorexia.
C) Individuals may eat to relieve depression or anxiety, which can lead to obesity. Overeating
will not lead to emaciation, hunger pangs, or anorexia.
D) Individuals may eat to relieve depression or anxiety, which can lead to obesity. Overeating
will not lead to emaciation, hunger pangs, or anorexia.
Page Ref: 791
Cognitive Level: Applying
Client Need: Health Promotion and Maintenance
Nursing Process: Implementation
Learning Outcome: 1. Describe the pathophysiology, etiology, clinical manifestations, and
direct and indirect causes of obesity.

37
Copyright © 2015 Pearson Education, Inc.
2) The nurse is caring for a 59-year-old client who is admitted to the hospital with a possible hip
fracture. Following the admission assessment, nurse determines that the client is obese. About
which disorders should the nurse teach the client that are often associated with obesity?
Select all that apply.
A) Stroke
B) Degenerative joint disease
C) Urinary retention
D) Mobility problems
E) Chronic cough
Answer: B, D
Explanation: A) A client who is overweight and/or obese is at an increased risk for degenerative
joint disease and functional and mobility problems. There is no relationship between obesity and
chronic cough or urinary retention. There may be some relationship indirectly to stroke.
B) A client who is overweight and/or obese is at an increased risk for degenerative joint disease
and functional and mobility problems. There is no relationship between obesity and chronic
cough or urinary retention. There may be some relationship indirectly to stroke.
C) A client who is overweight and/or obese is at an increased risk for degenerative joint disease
and functional and mobility problems. There is no relationship between obesity and chronic
cough or urinary retention. There may be some relationship indirectly to stroke.
D) A client who is overweight and/or obese is at an increased risk for degenerative joint disease
and functional and mobility problems. There is no relationship between obesity and chronic
cough or urinary retention. There may be some relationship indirectly to stroke.
E) A client who is overweight and/or obese is at an increased risk for degenerative joint disease
and functional and mobility problems. There is no relationship between obesity and chronic
cough or urinary retention. There may be some relationship indirectly to stroke.
Page Ref: 792
Cognitive Level: Applying
Client Need: Health Promotion and Maintenance
Nursing Process: Implementation
Learning Outcome: 2. Identify risk factors and prevention methods associated with obesity.

38
Copyright © 2015 Pearson Education, Inc.
3) The nurse is planning a teaching seminar for a group of young adult client who are at risk for
obesity. What should the nurse include in the program for this group?
A) There are drugs that are good to use to reduce weight.
B) Obesity puts the client at risk for anorexia nervosa.
C) Proper diet and exercise programs
D) The obese client will eventually be bulimic.
Answer: C
Explanation: A) The young adults who are at risk for obesity need education about changing
lifestyles and the importance of preventing obesity as opposed to treating it. Education should
include tips on eating healthy and exercising. Drugs used for obesity are not without risk.
Bulimia and anorexia nervosa are not eating disorders that are caused by obesity.
B) The young adults who are at risk for obesity need education about changing lifestyles and the
importance of preventing obesity as opposed to treating it. Education should include tips on
eating healthy and exercising. Drugs used for obesity are not without risk. Bulimia and anorexia
nervosa are not eating disorders that are caused by obesity.
C) The young adults who are at risk for obesity need education about changing lifestyles and the
importance of preventing obesity as opposed to treating it. Education should include tips on
eating healthy and exercising. Drugs used for obesity are not without risk. Bulimia and anorexia
nervosa are not eating disorders that are caused by obesity.
D) The young adults who are at risk for obesity need education about changing lifestyles and the
importance of preventing obesity as opposed to treating it. Education should include tips on
eating healthy and exercising. Drugs used for obesity are not without risk. Bulimia and anorexia
nervosa are not eating disorders that are caused by obesity.
Page Ref: 794
Cognitive Level: Applying
Client Need: Health Promotion and Maintenance
Nursing Process: Implementation
Learning Outcome: 3. Illustrate the nursing process in providing culturally competent care
across the life span for individuals with obesity.

39
Copyright © 2015 Pearson Education, Inc.
4) The nurse is working with a morbidly obese client who is seeking help with weight loss at a
bariatric clinic. The nurse is planning care and determines that which diagnosis is a priority for
this client?
A) Activity Intolerance
B) Disturbed Body Image
C) Defensive Coping
D) Constipation
Answer: A
Explanation: A) Along with diet, exercise is an important part of a weight loss program. A client
with morbid obesity has a sedentary lifestyle and will have activity intolerance. Disturbed Body
Image and Constipation may both be legitimate diagnoses, but Activity Intolerance is a greater
priority if the client is to lose weight. There is no evidence that this client exhibits defensive
coping.
B) Along with diet, exercise is an important part of a weight loss program. A client with morbid
obesity has a sedentary lifestyle and will have activity intolerance. Disturbed Body Image and
Constipation may both be legitimate diagnoses, but Activity Intolerance is a greater priority if
the client is to lose weight. There is no evidence that this client exhibits defensive coping.
C) Along with diet, exercise is an important part of a weight loss program. A client with morbid
obesity has a sedentary lifestyle and will have activity intolerance. Disturbed Body Image and
Constipation may both be legitimate diagnoses, but Activity Intolerance is a greater priority if
the client is to lose weight. There is no evidence that this client exhibits defensive coping.
D) Along with diet, exercise is an important part of a weight loss program. A client with morbid
obesity has a sedentary lifestyle and will have activity intolerance. Disturbed Body Image and
Constipation may both be legitimate diagnoses, but Activity Intolerance is a greater priority if
the client is to lose weight. There is no evidence that this client exhibits defensive coping.
Page Ref: 796
Cognitive Level: Analyzing
Client Need: Health Promotion and Maintenance
Nursing Process: Planning
Learning Outcome: 4. Formulate priority nursing diagnoses appropriate for an individual with
obesity.

40
Copyright © 2015 Pearson Education, Inc.
5) The nurse is planning care for a client with upper body obesity. What client teaching should
the nurse include in this client's care?
Select all that apply.
A) Oxygen consumption
B) Exercise
C) Injury prevention
D) Diet
E) Behavior modification
Answer: B, D, E
Explanation: A) Although many factors contribute to obesity, this condition always involves an
imbalance of kilocalorie consumption and energy expenditure. Patient education includes
exercise, diet, and behavior modification. Oxygen consumption and injury prevention are not
identified areas for client teaching regarding obesity.
B) Although many factors contribute to obesity, this condition always involves an imbalance of
kilocalorie consumption and energy expenditure. Patient education includes exercise, diet, and
behavior modification. Oxygen consumption and injury prevention are not identified areas for
client teaching regarding obesity.
C) Although many factors contribute to obesity, this condition always involves an imbalance of
kilocalorie consumption and energy expenditure. Patient education includes exercise, diet, and
behavior modification. Oxygen consumption and injury prevention are not identified areas for
client teaching regarding obesity.
D) Although many factors contribute to obesity, this condition always involves an imbalance of
kilocalorie consumption and energy expenditure. Patient education includes exercise, diet, and
behavior modification. Oxygen consumption and injury prevention are not identified areas for
client teaching regarding obesity.
E) Although many factors contribute to obesity, this condition always involves an imbalance of
kilocalorie consumption and energy expenditure. Patient education includes exercise, diet, and
behavior modification. Oxygen consumption and injury prevention are not identified areas for
client teaching regarding obesity.
Page Ref: 791-792
Cognitive Level: Applying
Client Need: Health Promotion and Maintenance
Nursing Process: Planning
Learning Outcome: 6. Plan evidence-based care for an individual with obesity and his or her
family in collaboration with other members of the healthcare team.

41
Copyright © 2015 Pearson Education, Inc.
6) The nurse is discussing postoperative care with a client scheduled for Roux-en-Y gastric
bypass surgery. Which client statement indicates that learning goals for this client have been
met?
A) "Maintaining protein intake will be a priority in my recovery diet."
B) "I will need to take daily vitamin and mineral supplements."
C) "I will initially take in only liquids, such as low-sugar juices."
D) "The foods I am allowed to eat gradually will be increased."
Answer: C
Explanation: A) Liquids are initially the only intake allowed after bariatric surgery. Sugary
beverages and fruit juices with added sugar should be avoided, as these products can promote
dumping syndrome. Vitamin and mineral supplements will be prescribed after the immediate
postoperative period.
B) Liquids are initially the only intake allowed after bariatric surgery. Sugary beverages and fruit
juices with added sugar should be avoided, as these products can promote dumping syndrome.
Vitamin and mineral supplements will be prescribed after the immediate postoperative period.
C) Liquids are initially the only intake allowed after bariatric surgery. Sugary beverages and fruit
juices with added sugar should be avoided, as these products can promote dumping syndrome.
Vitamin and mineral supplements will be prescribed after the immediate postoperative period.
D) Liquids are initially the only intake allowed after bariatric surgery. Sugary beverages and fruit
juices with added sugar should be avoided, as these products can promote dumping syndrome.
Vitamin and mineral supplements will be prescribed after the immediate postoperative period.
Page Ref: 795
Cognitive Level: Analyzing
Client Need: Physiological Integrity
Nursing Process: Evaluation
Learning Outcome: 7. Evaluate expected outcomes for an individual with obesity.

42
Copyright © 2015 Pearson Education, Inc.
7) The nurse is discussing a new diet order with a client who is seeking help to lose weight. The
client asks the nurse how to best balance the new diet. What should the nurse respond to this
client?
A) "Your diet should consist of 1,250-1,500 calories per day, with 15% of the calories being
sources of protein."
B) "Your diet should be consist of 750-1,000 calories per day, with less than 15% of the total
calories coming from fat."
C) "Your diet should simply cut 500 calories per day from your normal intake."
D) "Your diet should consist of 1,000-1,200 calories per day, with less than 15% of the total
calories coming from fat."
Answer: D
Explanation: A) Collaboration with a nutritionist helps clients to identify healthy foods that
appeal to them and that can make up a diet plan to create a daily 500- to 1,000-kcal deficit.
Ideally, the recommended diet should be low in kilocalories and fat, contain adequate nutrients
and minerals, and be high in dietary fiber. The client should eat regular meals with small
servings. A gradual, slow weight loss of no more than 1-2 lb/week is recommended. For most
individuals, this means a diet of 1,000-1,200 kcal/day for most women and 1,200-1,600 kcal/day
for men. Fewer than 1,200 kcal each day may lead to loss of lean tissue and nutritional
deficiencies.
B) Collaboration with a nutritionist helps clients to identify healthy foods that appeal to them and
that can make up a diet plan to create a daily 500- to 1,000-kcal deficit. Ideally, the
recommended diet should be low in kilocalories and fat, contain adequate nutrients and minerals,
and be high in dietary fiber. The client should eat regular meals with small servings. A gradual,
slow weight loss of no more than 1-2 lb/week is recommended. For most individuals, this means
a diet of 1,000-1,200 kcal/day for most women and 1,200-1,600 kcal/day for men. Fewer than
1,200 kcal each day may lead to loss of lean tissue and nutritional deficiencies.
C) Collaboration with a nutritionist helps clients to identify healthy foods that appeal to them and
that can make up a diet plan to create a daily 500- to 1,000-kcal deficit. Ideally, the
recommended diet should be low in kilocalories and fat, contain adequate nutrients and minerals,
and be high in dietary fiber. The client should eat regular meals with small servings. A gradual,
slow weight loss of no more than 1-2 lb/week is recommended. For most individuals, this means
a diet of 1,000-1,200 kcal/day for most women and 1,200-1,600 kcal/day for men. Fewer than
1,200 kcal each day may lead to loss of lean tissue and nutritional deficiencies.
D) Collaboration with a nutritionist helps clients to identify healthy foods that appeal to them
and that can make up a diet plan to create a daily 500- to 1,000-kcal deficit. Ideally, the
recommended diet should be low in kilocalories and fat, contain adequate nutrients and minerals,
and be high in dietary fiber. The client should eat regular meals with small servings. A gradual,
slow weight loss of no more than 1-2 lb/week is recommended. For most individuals, this means
a diet of 1,000-1,200 kcal/day for most women and 1,200-1,600 kcal/day for men. Fewer than
1,200 kcal each day may lead to loss of lean tissue and nutritional deficiencies.
Page Ref: 794
Cognitive Level: Applying
Client Need: Health Promotion and Maintenance
Nursing Process: Implementation
Learning Outcome: 5. Summarize therapies used by interdisciplinary teams in the collaborative
care of an individual with obesity.

43
Copyright © 2015 Pearson Education, Inc.
8) A nurse is working with a female client who is attempting to lose weight. The client admits
having difficulty being compliant with the diet prescribed by her doctor. Which suggestions
might assist the client in being compliant with the prescribed diet?
A) "Record your food intake so that you can see what and how much you are eating."
B) "Set aside small food reward when you meet a weight loss goal."
C) "Eat alone to reduce outside distractions that may cause you to stray from your diet."
D) "Allow at least 45 minutes to 1 hour to promote full enjoyment of your meal."
Answer: A
Explanation: A) Recording food intake, amount, location of eating, and situations that induce
eating often helps the dieter to gain self-control. Eating is a social activity. Talking with others
during mealtime promotes involvement. A meal should be slated to last only 20 minutes. Eating
longer can promote eating more. A small nonfood reward can serve as an incentive for working
toward a goal. Food rewards should be avoided.
B) Recording food intake, amount, location of eating, and situations that induce eating often
helps the dieter to gain self-control. Eating is a social activity. Talking with others during
mealtime promotes involvement. A meal should be slated to last only 20 minutes. Eating longer
can promote eating more. A small nonfood reward can serve as an incentive for working toward
a goal. Food rewards should be avoided.
C) Recording food intake, amount, location of eating, and situations that induce eating often
helps the dieter to gain self-control. Eating is a social activity. Talking with others during
mealtime promotes involvement. A meal should be slated to last only 20 minutes. Eating longer
can promote eating more. A small nonfood reward can serve as an incentive for working toward
a goal. Food rewards should be avoided.
D) Recording food intake, amount, location of eating, and situations that induce eating often
helps the dieter to gain self-control. Eating is a social activity. Talking with others during
mealtime promotes involvement. A meal should be slated to last only 20 minutes. Eating longer
can promote eating more. A small nonfood reward can serve as an incentive for working toward
a goal. Food rewards should be avoided.
Page Ref: 794
Cognitive Level: Applying
Client Need: Psychosocial Integrity
Nursing Process: Implementation
Learning Outcome: 6. Plan evidence-based care for an individual with obesity and his or her
family in collaboration with other members of the healthcare team.

44
Copyright © 2015 Pearson Education, Inc.
9) A nurse is caring for a client who weighs 209 pounds and is 1.67 meters tall. The nurse
calculates the body mass index (BMI) of this client as ________.
Answer: 34 kg/m2
Explanation:
Weight (209 lbs) height (1.67 meters)

(To convert lbs to kg) weight divided by 2.2 = kg


209 lb / 2.2 = 95 kg

(To calculate BMI using kg/meters squared) Weight (kg) divided by height (meters) squared
95 / (1.67 × 1.67)

95 / 2.79 = 34 BMI
Page Ref: 793
Cognitive Level: Understanding
Client Need: Health Promotion and Maintenance
Nursing Process: Implementation
Learning Outcome: 3. Illustrate the nursing process in providing culturally competent care
across the life span for individuals with obesity.

45
Copyright © 2015 Pearson Education, Inc.
10) A community health nurse is educating a group of adults about obesity and weight
management. Which statements will the nurse include regarding upper body obesity?
Select all that apply.
A) "This is also called peripheral obesity."
B) "It is when the waist-to-hip ratio in men is greater than 0.8 in men or greater than 1 in
women."
C) "It is associated with a greater risk of hypertension."
D) "Young women tend to have more intra-abdominal fat than men."
E) "Postmenopausal women tend to have this."
Answer: C, E
Explanation: A) Upper body obesity (also called central obesity) is identified by a waist-to-hip
ratio of greater than 1 in men or 0.8 in women. Upper body obesity is associated with a greater
risk of complications such as hypertension, abnormal blood lipid levels, heart disease, stroke,
and elevated insulin levels. Men tend to have more intra-abdominal fat than women, although
women develop a central fat distribution pattern after menopause.
B) Upper body obesity (also called central obesity) is identified by a waist-to-hip ratio of greater
than 1 in men or 0.8 in women. Upper body obesity is associated with a greater risk of
complications such as hypertension, abnormal blood lipid levels, heart disease, stroke, and
elevated insulin levels. Men tend to have more intra-abdominal fat than women, although women
develop a central fat distribution pattern after menopause.
C) Upper body obesity (also called central obesity) is identified by a waist-to-hip ratio of greater
than 1 in men or 0.8 in women. Upper body obesity is associated with a greater risk of
complications such as hypertension, abnormal blood lipid levels, heart disease, stroke, and
elevated insulin levels. Men tend to have more intra-abdominal fat than women, although women
develop a central fat distribution pattern after menopause.
D) Upper body obesity (also called central obesity) is identified by a waist-to-hip ratio of greater
than 1 in men or 0.8 in women. Upper body obesity is associated with a greater risk of
complications such as hypertension, abnormal blood lipid levels, heart disease, stroke, and
elevated insulin levels. Men tend to have more intra-abdominal fat than women, although women
develop a central fat distribution pattern after menopause.
E) Upper body obesity (also called central obesity) is identified by a waist-to-hip ratio of greater
than 1 in men or 0.8 in women. Upper body obesity is associated with a greater risk of
complications such as hypertension, abnormal blood lipid levels, heart disease, stroke, and
elevated insulin levels. Men tend to have more intra-abdominal fat than women, although women
develop a central fat distribution pattern after menopause.
Page Ref: 791
Cognitive Level: Understanding
Client Need: Health Promotion and Maintenance
Nursing Process: Implementation
Learning Outcome: 3. Illustrate the nursing process in providing culturally competent care
across the life span for individuals with obesity.

46
Copyright © 2015 Pearson Education, Inc.
Exemplar 12.5 Osteoporosis

1) The nurse is assessing a 70-year-old postmenopausal client. Which question should the nurse
ask to assess for signs of osteoporosis?
A) "Have you experienced any palpitations?"
B) "Are you having any low back pain?"
C) "Are you having problems with swelling in your feet?"
D) "Is constipation a problem for you?"
Answer: B
Explanation: A) A client with osteoporosis will often present with low back pain as well as a
decrease in height. Palpitations, constipation, and swelling are not early signs of osteoporosis.
B) A client with osteoporosis will often present with low back pain as well as a decrease in
height. Palpitations, constipation, and swelling are not early signs of osteoporosis.
C) A client with osteoporosis will often present with low back pain as well as a decrease in
height. Palpitations, constipation, and swelling are not early signs of osteoporosis.
D) A client with osteoporosis will often present with low back pain as well as a decrease in
height. Palpitations, constipation, and swelling are not early signs of osteoporosis.
Page Ref: 800
Cognitive Level: Applying
Client Need: Health Promotion and Maintenance
Nursing Process: Assessment
Learning Outcome: 1. Describe the pathophysiology, etiology, clinical manifestations, and
direct and indirect causes of osteoporosis.

47
Copyright © 2015 Pearson Education, Inc.
2) A nurse is conducting a health history on an older woman who is a new client in a medical
practice. Which assessment finding places the client at risk for osteoporosis?
A) The client is obese and has hip pain with ambulation.
B) The client has been taking corticosteroids for 10 years because of chronic obstructive
pulmonary disease (COPD).
C) The client eats 3-5 servings of shrimp and liver per week.
D) The client states she drinks three glasses of skim milk daily.
Answer: B
Explanation: A) Long-time use of corticosteroids is a risk factor for developing osteoporosis.
Obesity is not a risk factor for osteoporosis but predisposes the woman to osteoarthritis. Skim
milk is a good source of calcium and vitamin D, which prevents or slows osteoporosis. A diet
rich in shellfish and organ meats is high in purine, which may predispose the client to gout.
B) Long-time use of corticosteroids is a risk factor for developing osteoporosis. Obesity is not a
risk factor for osteoporosis but predisposes the woman to osteoarthritis. Skim milk is a good
source of calcium and vitamin D, which prevents or slows osteoporosis. A diet rich in shellfish
and organ meats is high in purine, which may predispose the client to gout.
C) Long-time use of corticosteroids is a risk factor for developing osteoporosis. Obesity is not a
risk factor for osteoporosis but predisposes the woman to osteoarthritis. Skim milk is a good
source of calcium and vitamin D, which prevents or slows osteoporosis. A diet rich in shellfish
and organ meats is high in purine, which may predispose the client to gout.
D) Long-time use of corticosteroids is a risk factor for developing osteoporosis. Obesity is not a
risk factor for osteoporosis but predisposes the woman to osteoarthritis. Skim milk is a good
source of calcium and vitamin D, which prevents or slows osteoporosis. A diet rich in shellfish
and organ meats is high in purine, which may predispose the client to gout.
Page Ref: 799
Cognitive Level: Analyzing
Client Need: Health Promotion and Maintenance
Nursing Process: Assessment
Learning Outcome: 2. Identify risk factors and prevention methods associated with osteoporosis.

48
Copyright © 2015 Pearson Education, Inc.
3) The nurse is planning care for a 66-year-old client who is in one of the high-risk groups for
developing osteoporosis. Which interventions will decrease the client's risk of developing this
health problem?
Select all that apply.
A) Increasing the intake of beverages that contain phosphorous
B) Isometric exercise for at least 30 minutes three times per week
C) Weight-bearing exercises such as walking
D) Having a yearly bone mineral density (BMD) test
E) A diet with adequate amounts of calcium and vitamin D
Answer: C, E
Explanation: A) Patients, especially women, are encouraged to include adequate calcium and
vitamin D in their diets to prevent osteoporotic fracture. A BMD test is used to determine bone
strength and risk for osteoporotic fracture. A BMD test is recommended for women under age 65
with risk factors, all women over 65, and after a fracture, but not yearly. BMD tests are
sometimes repeated to monitor effects from medications used to treat osteoporosis. Measures to
prevent or treat osteoporosis include limiting the intake of beverages containing alcohol,
caffeine, and phosphorus. Regular weight-bearing exercise, such as walking, slows bone loss.
Isometric exercises are not effective against osteoporosis.
B) Patients, especially women, are encouraged to include adequate calcium and vitamin D in
their diets to prevent osteoporotic fracture. A BMD test is used to determine bone strength and
risk for osteoporotic fracture. A BMD test is recommended for women under age 65 with risk
factors, all women over 65, and after a fracture, but not yearly. BMD tests are sometimes
repeated to monitor effects from medications used to treat osteoporosis. Measures to prevent or
treat osteoporosis include limiting the intake of beverages containing alcohol, caffeine, and
phosphorus. Regular weight-bearing exercise, such as walking, slows bone loss. Isometric
exercises are not effective against osteoporosis.
C) Patients, especially women, are encouraged to include adequate calcium and vitamin D in
their diets to prevent osteoporotic fracture. A BMD test is used to determine bone strength and
risk for osteoporotic fracture. A BMD test is recommended for women under age 65 with risk
factors, all women over 65, and after a fracture, but not yearly. BMD tests are sometimes
repeated to monitor effects from medications used to treat osteoporosis. Measures to prevent or
treat osteoporosis include limiting the intake of beverages containing alcohol, caffeine, and
phosphorus. Regular weight-bearing exercise, such as walking, slows bone loss. Isometric
exercises are not effective against osteoporosis.
D) Patients, especially women, are encouraged to include adequate calcium and vitamin D in
their diets to prevent osteoporotic fracture. A BMD test is used to determine bone strength and
risk for osteoporotic fracture. A BMD test is recommended for women under age 65 with risk
factors, all women over 65, and after a fracture, but not yearly. BMD tests are sometimes
repeated to monitor effects from medications used to treat osteoporosis. Measures to prevent or
treat osteoporosis include limiting the intake of beverages containing alcohol, caffeine, and
phosphorus. Regular weight-bearing exercise, such as walking, slows bone loss. Isometric
exercises are not effective against osteoporosis.

49
Copyright © 2015 Pearson Education, Inc.
E) Patients, especially women, are encouraged to include adequate calcium and vitamin D in
their diets to prevent osteoporotic fracture. A BMD test is used to determine bone strength and
risk for osteoporotic fracture. A BMD test is recommended for women under age 65 with risk
factors, all women over 65, and after a fracture, but not yearly. BMD tests are sometimes
repeated to monitor effects from medications used to treat osteoporosis. Measures to prevent or
treat osteoporosis include limiting the intake of beverages containing alcohol, caffeine, and
phosphorus. Regular weight-bearing exercise, such as walking, slows bone loss. Isometric
exercises are not effective against osteoporosis.
Page Ref: 799
Cognitive Level: Applying
Client Need: Health Promotion and Maintenance
Nursing Process: Implementation
Learning Outcome: 3. Illustrate the nursing process in providing culturally competent care
across the life span for individuals with osteoporosis.

4) The nurse identifies the diagnosis of Imbalanced Nutrition as appropriate for a client with
osteoporosis. Which client statement did the nurse use to identify this diagnosis?
A) "I have removed all scatter rugs from my home."
B) "I frequently take long walks in the sun."
C) "My pain is relieved by Tylenol."
D) "I am allergic to dairy products."
Answer: D
Explanation: A) The client states that she is allergic to dairy products and therefore may not take
in much calcium, so focusing on diet would be a priority for this client. The statements about
taking long walks, removing scatter rugs, and taking acetaminophen (Tylenol) for pain would not
elicit the nursing diagnosis Imbalanced Nutrition.
B) The client states that she is allergic to dairy products and therefore may not take in much
calcium, so focusing on diet would be a priority for this client. The statements about taking long
walks, removing scatter rugs, and taking acetaminophen (Tylenol) for pain would not elicit the
nursing diagnosis Imbalanced Nutrition.
C) The client states that she is allergic to dairy products and therefore may not take in much
calcium, so focusing on diet would be a priority for this client. The statements about taking long
walks, removing scatter rugs, and taking acetaminophen (Tylenol) for pain would not elicit the
nursing diagnosis Imbalanced Nutrition.
D) The client states that she is allergic to dairy products and therefore may not take in much
calcium, so focusing on diet would be a priority for this client. The statements about taking long
walks, removing scatter rugs, and taking acetaminophen (Tylenol) for pain would not elicit the
nursing diagnosis Imbalanced Nutrition.
Page Ref: 802
Cognitive Level: Analyzing
Client Need: Physiological Integrity
Nursing Process: Planning
Learning Outcome: 4. Formulate priority nursing diagnoses appropriate for an individual with
osteoporosis.

50
Copyright © 2015 Pearson Education, Inc.
5) A client recently diagnosed with osteoporosis asks what can be done to slow the progress of
the disease. Which intervention would be the most beneficial for this client?
A) Decreasing the amount of calcium in the client's diet
B) Providing the client with assisted range of motion exercising twice daily
C) An exercise plan that includes weight-bearing activities
D) Protecting the client's bones with strict bed rest
Answer: C
Explanation: A) Osteoporosis is a demineralization of the bone in which calcium leaves the
bone matrix. One causative factor is lack of weight-bearing activity. Weight bearing helps to
move calcium back into the bone, thereby strengthening them. A standard intervention for those
attempting to prevent or reverse osteoporosis is beginning an exercise plan that includes weight-
bearing activities. Strict bed rest may well make the osteoporosis worse because there is no
weight-bearing activity. Calcium in the diet is increased with osteoporosis. Assisted range of
motion exercises are not weight-bearing and do not help delay or reverse osteoporosis.
B) Osteoporosis is a demineralization of the bone in which calcium leaves the bone matrix. One
causative factor is lack of weight-bearing activity. Weight bearing helps to move calcium back
into the bone, thereby strengthening them. A standard intervention for those attempting to
prevent or reverse osteoporosis is beginning an exercise plan that includes weight-bearing
activities. Strict bed rest may well make the osteoporosis worse because there is no weight-
bearing activity. Calcium in the diet is increased with osteoporosis. Assisted range of motion
exercises are not weight-bearing and do not help delay or reverse osteoporosis.
C) Osteoporosis is a demineralization of the bone in which calcium leaves the bone matrix. One
causative factor is lack of weight-bearing activity. Weight bearing helps to move calcium back
into the bone, thereby strengthening them. A standard intervention for those attempting to
prevent or reverse osteoporosis is beginning an exercise plan that includes weight-bearing
activities. Strict bed rest may well make the osteoporosis worse because there is no weight-
bearing activity. Calcium in the diet is increased with osteoporosis. Assisted range of motion
exercises are not weight-bearing and do not help delay or reverse osteoporosis.
D) Osteoporosis is a demineralization of the bone in which calcium leaves the bone matrix. One
causative factor is lack of weight-bearing activity. Weight bearing helps to move calcium back
into the bone, thereby strengthening them. A standard intervention for those attempting to
prevent or reverse osteoporosis is beginning an exercise plan that includes weight-bearing
activities. Strict bed rest may well make the osteoporosis worse because there is no weight-
bearing activity. Calcium in the diet is increased with osteoporosis. Assisted range of motion
exercises are not weight-bearing and do not help delay or reverse osteoporosis.
Page Ref: 800
Cognitive Level: Analyzing
Client Need: Health Promotion and Maintenance
Nursing Process: Implementation
Learning Outcome: 6. Plan evidence-based care for an individual with osteoporosis and his or
her family in collaboration with other members of the healthcare team.

51
Copyright © 2015 Pearson Education, Inc.
6) A 78-year-old female client with osteoporosis has a history of falls and dementia. Which
interventions will best aid in meeting an outcome goal for injury prevention?
A) Using furniture as obstacles to keep the client in the bed
B) Keeping the bed in the lowest position
C) Keeping a nightlight on in the room
D) The use of wrist restraints
Answer: B
Explanation: A) Keeping the bed in the lowest position will reduce the incidence of injury
should the client attempt to get up. The use of restraints could increase the incidence of injury.
Using the furniture as an obstacle could cause injury if the client is able to get up. A nightlight is
useful but is not the best means to prevent injury.
B) Keeping the bed in the lowest position will reduce the incidence of injury should the client
attempt to get up. The use of restraints could increase the incidence of injury. Using the furniture
as an obstacle could cause injury if the client is able to get up. A nightlight is useful but is not the
best means to prevent injury.
C) Keeping the bed in the lowest position will reduce the incidence of injury should the client
attempt to get up. The use of restraints could increase the incidence of injury. Using the furniture
as an obstacle could cause injury if the client is able to get up. A nightlight is useful but is not the
best means to prevent injury.
D) Keeping the bed in the lowest position will reduce the incidence of injury should the client
attempt to get up. The use of restraints could increase the incidence of injury. Using the furniture
as an obstacle could cause injury if the client is able to get up. A nightlight is useful but is not the
best means to prevent injury.
Page Ref: 803
Cognitive Level: Analyzing
Client Need: Safe and Effective Care Environment
Nursing Process: Planning
Learning Outcome: 7. Evaluate expected outcomes for an individual with osteoporosis.

52
Copyright © 2015 Pearson Education, Inc.
7) A 30-year-old female is concerned because the healthcare provider states that the client is
demonstrating signs consistent with early onset of osteoporosis. What should the nurse explain to
assist the client at this time?
A) Suggest that the client stop all physical activity.
B) Recommend reducing the intake of diary in the diet.
C) Instruct on a diet with an adequate intake of calcium and vitamin D.
D) Discuss the use of estrogen replacement therapy.
Answer: C
Explanation: A) Walking and weight-bearing exercise help prevent the onset of osteoporosis. A
diet that includes dairy products will have calcium and vitamin D, which are nutrients needed to
prevent the onset of the disorder. The client is 30 years old and most likely does not need
estrogen replacement therapy at this time.
B) Walking and weight-bearing exercise help prevent the onset of osteoporosis. A diet that
includes dairy products will have calcium and vitamin D, which are nutrients needed to prevent
the onset of the disorder. The client is 30 years old and most likely does not need estrogen
replacement therapy at this time.
C) Walking and weight-bearing exercise help prevent the onset of osteoporosis. A diet that
includes dairy products will have calcium and vitamin D, which are nutrients needed to prevent
the onset of the disorder. The client is 30 years old and most likely does not need estrogen
replacement therapy at this time.
D) Walking and weight-bearing exercise help prevent the onset of osteoporosis. A diet that
includes dairy products will have calcium and vitamin D, which are nutrients needed to prevent
the onset of the disorder. The client is 30 years old and most likely does not need estrogen
replacement therapy at this time.
Page Ref: 802
Cognitive Level: Applying
Client Need: Health Promotion and Maintenance
Nursing Process: Implementation
Learning Outcome: 5. Summarize therapies used by interdisciplinary teams in the collaborative
care of an individual with osteoporosis.

53
Copyright © 2015 Pearson Education, Inc.
8) A 60-year-old client is concerned about the development of osteoporosis and wants to begin
preventative activities. What response by the nurse is the most appropriate?
A) "You should first determine if you are at risk for the development of osteoporosis."
B) "After menopause, the decline is too rapid to begin preventative interventions."
C) "Weight-bearing exercise and calcium supplements are helpful in the prevention of
osteoporosis."
D) "Hormone replacement therapy should be initiated as soon as possible."
Answer: C
Explanation: A) Osteoporosis risk factors increase after menopause. Preventative activities
include implementing weight-bearing exercise and beginning calcium supplements. It is not too
late to begin prevention activities. Without additional information, it is not possible to determine
if the client is a candidate for hormone replacement therapy. Generally, hormone replacement
therapy is limited to those women who are experiencing vasomotor manifestations. The client in
the scenario has two risk factors presented. Although a full analysis would be beneficial, it does
not answer the client's request for information.
B) Osteoporosis risk factors increase after menopause. Preventative activities include
implementing weight-bearing exercise and beginning calcium supplements. It is not too late to
begin prevention activities. Without additional information, it is not possible to determine if the
client is a candidate for hormone replacement therapy. Generally, hormone replacement therapy
is limited to those women who are experiencing vasomotor manifestations. The client in the
scenario has two risk factors presented. Although a full analysis would be beneficial, it does not
answer the client's request for information.
C) Osteoporosis risk factors increase after menopause. Preventative activities include
implementing weight-bearing exercise and beginning calcium supplements. It is not too late to
begin prevention activities. Without additional information, it is not possible to determine if the
client is a candidate for hormone replacement therapy. Generally, hormone replacement therapy
is limited to those women who are experiencing vasomotor manifestations. The client in the
scenario has two risk factors presented. Although a full analysis would be beneficial, it does not
answer the client's request for information.
D) Osteoporosis risk factors increase after menopause. Preventative activities include
implementing weight-bearing exercise and beginning calcium supplements. It is not too late to
begin prevention activities. Without additional information, it is not possible to determine if the
client is a candidate for hormone replacement therapy. Generally, hormone replacement therapy
is limited to those women who are experiencing vasomotor manifestations. The client in the
scenario has two risk factors presented. Although a full analysis would be beneficial, it does not
answer the client's request for information.
Page Ref: 803
Cognitive Level: Applying
Client Need: Health Promotion and Maintenance
Nursing Process: Implementation
Learning Outcome: 2. Identify risk factors and prevention methods associated with osteoporosis.

54
Copyright © 2015 Pearson Education, Inc.
9) The nurse is caring for an elderly client who is taking calcium for the treatment of
osteoporosis. Which statements will the nurse include when educating the client about this
medication?
Select all that apply.
A) "The most common adverse effect is hypercalcemia caused by taking too much of the
supplement."
B) "Oral calcium supplements are best taken on an empty stomach."
C) "Adults 50 years of age and over should obtain at least 500-750 mg per day of elemental
calcium."
D) "If you have a condition called ventricular fibrillation, this medication might help."
E) "Report symptoms of weakness, increased urination, and thirst."
Answer: A, E
Explanation: A) Calcium gluconate and other calcium compounds are used to treat and prevent
osteoporosis. Oral calcium supplements are best taken with meals or within 1 hour following
meals. It is recommended that adults 50 years of age and over obtain at least 1,000-1,200 mg per
day of elemental calcium. The most common adverse effect is hypercalcemia caused by taking
too much of the supplement. Symptoms include lethargy, drowsiness, weakness, headache,
anorexia, nausea and vomiting, increased urination, and thirst. Calcium supplementation is
contraindicated in clients with ventricular fibrillation.
B) Calcium gluconate and other calcium compounds are used to treat and prevent osteoporosis.
Oral calcium supplements are best taken with meals or within 1 hour following meals. It is
recommended that adults 50 years of age and over obtain at least 1,000-1,200 mg per day of
elemental calcium. The most common adverse effect is hypercalcemia caused by taking too
much of the supplement. Symptoms include lethargy, drowsiness, weakness, headache, anorexia,
nausea and vomiting, increased urination, and thirst. Calcium supplementation is contraindicated
in clients with ventricular fibrillation.
C) Calcium gluconate and other calcium compounds are used to treat and prevent osteoporosis.
Oral calcium supplements are best taken with meals or within 1 hour following meals. It is
recommended that adults 50 years of age and over obtain at least 1,000-1,200 mg per day of
elemental calcium. The most common adverse effect is hypercalcemia caused by taking too
much of the supplement. Symptoms include lethargy, drowsiness, weakness, headache, anorexia,
nausea and vomiting, increased urination, and thirst. Calcium supplementation is contraindicated
in clients with ventricular fibrillation.
D) Calcium gluconate and other calcium compounds are used to treat and prevent osteoporosis.
Oral calcium supplements are best taken with meals or within 1 hour following meals. It is
recommended that adults 50 years of age and over obtain at least 1,000-1,200 mg per day of
elemental calcium. The most common adverse effect is hypercalcemia caused by taking too
much of the supplement. Symptoms include lethargy, drowsiness, weakness, headache, anorexia,
nausea and vomiting, increased urination, and thirst. Calcium supplementation is contraindicated
in clients with ventricular fibrillation.

55
Copyright © 2015 Pearson Education, Inc.
E) Calcium gluconate and other calcium compounds are used to treat and prevent osteoporosis.
Oral calcium supplements are best taken with meals or within 1 hour following meals. It is
recommended that adults 50 years of age and over obtain at least 1,000-1,200 mg per day of
elemental calcium. The most common adverse effect is hypercalcemia caused by taking too
much of the supplement. Symptoms include lethargy, drowsiness, weakness, headache, anorexia,
nausea and vomiting, increased urination, and thirst. Calcium supplementation is contraindicated
in clients with ventricular fibrillation.
Page Ref: 801
Cognitive Level: Understanding
Client Need: Physiological Integrity
Nursing Process: Implementation
Learning Outcome: 3. Illustrate the nursing process in providing culturally competent care
across the life span for individuals with osteoporosis.

56
Copyright © 2015 Pearson Education, Inc.
10) A nurse is educating a group of adults about the risks for osteoporosis. Which statements will
the nurse include when discussing the use of alcohol and cigarettes and their link to
osteoporosis?
Select all that apply.
A) "Smoking decreases nerve supply to the bones."
B) "Nicotine increases calcium absorption, leading to decreased bone density."
C) "Moderate alcohol consumption in postmenopausal women actually may increase bone
mineral content."
D) "Alcohol has a direct toxic effect on osteoclast activity, suppressing bone formation."
E) "Heavy alcohol use may be associated with nutritional deficiencies that contribute to
osteoporosis."
Answer: C, E
Explanation: A) Both cigarette smoking and excess alcohol intake are risk factors for
osteoporosis. Smoking decreases the blood supply to bones, and nicotine slows the production of
osteoblasts and impairs the absorption of calcium, contributing to decreased bone density.
Alcohol has a direct toxic effect on osteoblast activity, suppressing bone formation during
periods of alcohol intoxication. In addition, heavy alcohol use may be associated with nutritional
deficiencies that contribute to osteoporosis. Interestingly, moderate alcohol consumption in
postmenopausal women actually may increase bone mineral content, possibly by increasing
levels of estrogen and calcitonin.
B) Both cigarette smoking and excess alcohol intake are risk factors for osteoporosis. Smoking
decreases the blood supply to bones, and nicotine slows the production of osteoblasts and
impairs the absorption of calcium, contributing to decreased bone density. Alcohol has a direct
toxic effect on osteoblast activity, suppressing bone formation during periods of alcohol
intoxication. In addition, heavy alcohol use may be associated with nutritional deficiencies that
contribute to osteoporosis. Interestingly, moderate alcohol consumption in postmenopausal
women actually may increase bone mineral content, possibly by increasing levels of estrogen and
calcitonin.
C) Both cigarette smoking and excess alcohol intake are risk factors for osteoporosis. Smoking
decreases the blood supply to bones, and nicotine slows the production of osteoblasts and
impairs the absorption of calcium, contributing to decreased bone density. Alcohol has a direct
toxic effect on osteoblast activity, suppressing bone formation during periods of alcohol
intoxication. In addition, heavy alcohol use may be associated with nutritional deficiencies that
contribute to osteoporosis. Interestingly, moderate alcohol consumption in postmenopausal
women actually may increase bone mineral content, possibly by increasing levels of estrogen and
calcitonin.
D) Both cigarette smoking and excess alcohol intake are risk factors for osteoporosis. Smoking
decreases the blood supply to bones, and nicotine slows the production of osteoblasts and
impairs the absorption of calcium, contributing to decreased bone density. Alcohol has a direct
toxic effect on osteoblast activity, suppressing bone formation during periods of alcohol
intoxication. In addition, heavy alcohol use may be associated with nutritional deficiencies that
contribute to osteoporosis. Interestingly, moderate alcohol consumption in postmenopausal
women actually may increase bone mineral content, possibly by increasing levels of estrogen and
calcitonin.

57
Copyright © 2015 Pearson Education, Inc.
E) Both cigarette smoking and excess alcohol intake are risk factors for osteoporosis. Smoking
decreases the blood supply to bones, and nicotine slows the production of osteoblasts and
impairs the absorption of calcium, contributing to decreased bone density. Alcohol has a direct
toxic effect on osteoblast activity, suppressing bone formation during periods of alcohol
intoxication. In addition, heavy alcohol use may be associated with nutritional deficiencies that
contribute to osteoporosis. Interestingly, moderate alcohol consumption in postmenopausal
women actually may increase bone mineral content, possibly by increasing levels of estrogen and
calcitonin.
Page Ref: 800
Cognitive Level: Analyzing
Client Need: Health Promotion and Maintenance
Nursing Process: Implementation
Learning Outcome: 2. Identify risk factors and prevention methods associated with osteoporosis.

Exemplar 12.6 Thyroid Disease

1) The nurse suspects that a client is experiencing hypothyroidism. Which question should the
nurse ask to assess for this disorder?
A) "Is your skin often clammy?"
B) "Do you have brown, shiny patches on your legs?"
C) "Is your skin smooth?"
D) "Is your skin rough and dry?"
Answer: D
Explanation: A) The client experiencing hypothyroidism has rough, dry skin. Smooth skin is
associated with hyperthyroidism. Cool, clammy skin is found in clients with low blood sugar.
Brown, shiny patches on the lower extremities are associated with poor circulation.
B) The client experiencing hypothyroidism has rough, dry skin. Smooth skin is associated with
hyperthyroidism. Cool, clammy skin is found in clients with low blood sugar. Brown, shiny
patches on the lower extremities are associated with poor circulation.
C) The client experiencing hypothyroidism has rough, dry skin. Smooth skin is associated with
hyperthyroidism. Cool, clammy skin is found in clients with low blood sugar. Brown, shiny
patches on the lower extremities are associated with poor circulation.
D) The client experiencing hypothyroidism has rough, dry skin. Smooth skin is associated with
hyperthyroidism. Cool, clammy skin is found in clients with low blood sugar. Brown, shiny
patches on the lower extremities are associated with poor circulation.
Page Ref: 806
Cognitive Level: Applying
Client Need: Health Promotion and Maintenance
Nursing Process: Assessment
Learning Outcome: 1. Describe the pathophysiology, etiology, clinical manifestations, and
direct and indirect causes of thyroid disease.

58
Copyright © 2015 Pearson Education, Inc.
2) The nurse is preparing an education session for nurses who work in an endocrinology clinic
that cares for the older clients. Which characteristic of hypothyroidism should the nurse include
for this group?
A) Thyroid hormone is often increased in the elderly.
B) Symptoms of hypothyroidism in the elderly are often confused with symptoms of aging.
C) Hypothyroidism in the elderly is a congenital disease.
D) The elderly client with hypothyroidism presents with pitting edema.
Answer: B
Explanation: A) The nurse educator must emphasize that the risk with elderly clients is that the
diagnosis of hypothyroidism can be missed as symptoms of aging may mask those of the disease.
Not all hypothyroidism is congenital, and it is inaccurate to state that older clients develop the
disease due to congenital defects. Thyroid hormone is decreased in all clients with
hypothyroidism. The adult client will present with non-pitting edema.
B) The nurse educator must emphasize that the risk with elderly clients is that the diagnosis of
hypothyroidism can be missed as symptoms of aging may mask those of the disease. Not all
hypothyroidism is congenital, and it is inaccurate to state that older clients develop the disease
due to congenital defects. Thyroid hormone is decreased in all clients with hypothyroidism. The
adult client will present with non-pitting edema.
C) The nurse educator must emphasize that the risk with elderly clients is that the diagnosis of
hypothyroidism can be missed as symptoms of aging may mask those of the disease. Not all
hypothyroidism is congenital, and it is inaccurate to state that older clients develop the disease
due to congenital defects. Thyroid hormone is decreased in all clients with hypothyroidism. The
adult client will present with non-pitting edema.
D) The nurse educator must emphasize that the risk with elderly clients is that the diagnosis of
hypothyroidism can be missed as symptoms of aging may mask those of the disease. Not all
hypothyroidism is congenital, and it is inaccurate to state that older clients develop the disease
due to congenital defects. Thyroid hormone is decreased in all clients with hypothyroidism. The
adult client will present with non-pitting edema.
Page Ref: 812
Cognitive Level: Applying
Client Need: Health Promotion and Maintenance
Nursing Process: Planning
Learning Outcome: 2. Identify risk factors and prevention methods associated with thyroid
disease.

59
Copyright © 2015 Pearson Education, Inc.
3) A nurse is caring for a 35-year-old female client who was recently diagnosed with
hypothyroidism. After reviewing the nursing admission assessment, on which documented
findings should the nurse plan care for this client?
Select all that apply.
A) Hypothermia
B) Hot flashes
C) Nausea
D) Fatigue
E) Tachycardia
Answer: A, D
Explanation: A) Symptoms of hypothyroidism include fatigue, increased need for sleep,
bradycardia, cold intolerance, ataxia and balance difficulties, and hypothermia. Hot flashes,
tachycardia, and nausea are not symptoms of hypothyroidism.
B) Symptoms of hypothyroidism include fatigue, increased need for sleep, bradycardia, cold
intolerance, ataxia and balance difficulties, and hypothermia. Hot flashes, tachycardia, and
nausea are not symptoms of hypothyroidism.
C) Symptoms of hypothyroidism include fatigue, increased need for sleep, bradycardia, cold
intolerance, ataxia and balance difficulties, and hypothermia. Hot flashes, tachycardia, and
nausea are not symptoms of hypothyroidism.
D) Symptoms of hypothyroidism include fatigue, increased need for sleep, bradycardia, cold
intolerance, ataxia and balance difficulties, and hypothermia. Hot flashes, tachycardia, and
nausea are not symptoms of hypothyroidism.
E) Symptoms of hypothyroidism include fatigue, increased need for sleep, bradycardia, cold
intolerance, ataxia and balance difficulties, and hypothermia. Hot flashes, tachycardia, and
nausea are not symptoms of hypothyroidism.
Page Ref: 813
Cognitive Level: Applying
Client Need: Physiological Integrity
Nursing Process: Planning
Learning Outcome: 3. Illustrate the nursing process in providing culturally competent care
across the life span for individuals with thyroid disease.

60
Copyright © 2015 Pearson Education, Inc.
4) The nurse is providing care for a 20-year-old client with exophthalmos. Which nursing
diagnosis would be the most appropriate for this clinical finding?
A) Disturbed Body Image
B) Ineffective Coping
C) Risk for Injury
D) Activity Intolerance
Answer: A
Explanation: A) Exophthalmos is a clinical manifestation associated with hyperthyroidism and
may be a problem for a young client. The nurse would plan to assess self-esteem and make
appropriate referrals. Activity intolerance and risk for injury are not particular to this medical
diagnosis. The client's ability to cope could be an issue, but it would probably stem from the
disturbed body image.
B) Exophthalmos is a clinical manifestation associated with hyperthyroidism and may be a
problem for a young client. The nurse would plan to assess self-esteem and make appropriate
referrals. Activity intolerance and risk for injury are not particular to this medical diagnosis. The
client's ability to cope could be an issue, but it would probably stem from the disturbed body
image.
C) Exophthalmos is a clinical manifestation associated with hyperthyroidism and may be a
problem for a young client. The nurse would plan to assess self-esteem and make appropriate
referrals. Activity intolerance and risk for injury are not particular to this medical diagnosis. The
client's ability to cope could be an issue, but it would probably stem from the disturbed body
image.
D) Exophthalmos is a clinical manifestation associated with hyperthyroidism and may be a
problem for a young client. The nurse would plan to assess self-esteem and make appropriate
referrals. Activity intolerance and risk for injury are not particular to this medical diagnosis. The
client's ability to cope could be an issue, but it would probably stem from the disturbed body
image.
Page Ref: 810
Cognitive Level: Analyzing
Client Need: Psychosocial Integrity
Nursing Process: Planning
Learning Outcome: 4. Formulate priority nursing diagnoses appropriate for an individual with
thyroid disease.

61
Copyright © 2015 Pearson Education, Inc.
5) The home health nurse is visiting a 3-month-old with congenital hypothyroidism who has
been prescribed a daily dose of thyroxine. Due to digestive issues, the baby is on soy formula
and is at the 50th percentile for height and weight. What should the nurse plan to teach the
mother?
A) Parents may stop the thyroxine as long as the baby remains in the 50th percentile for height
and weight.
B) Soy-based formula can interfere with the absorption of thyroxine.
C) Dairy-based formula is contraindicated when an infant is taking thyroxine.
D) As long as the baby is growing along the same growth curve, no interventions are necessary.
Answer: B
Explanation: A) Soybeans may block thyroid hormone synthesis if consumed in sufficient
quantities, which could occur in babies who are given soy-based formula. Dairy-based formula
does not interfere with thyroxine. Frequent checks must be done to assess the baby for
inadequate growth. To ensure adequate growth and development, the hormone must be taken
throughout life.
B) Soybeans may block thyroid hormone synthesis if consumed in sufficient quantities, which
could occur in babies who are given soy-based formula. Dairy-based formula does not interfere
with thyroxine. Frequent checks must be done to assess the baby for inadequate growth. To
ensure adequate growth and development, the hormone must be taken throughout life.
C) Soybeans may block thyroid hormone synthesis if consumed in sufficient quantities, which
could occur in babies who are given soy-based formula. Dairy-based formula does not interfere
with thyroxine. Frequent checks must be done to assess the baby for inadequate growth. To
ensure adequate growth and development, the hormone must be taken throughout life.
D) Soybeans may block thyroid hormone synthesis if consumed in sufficient quantities, which
could occur in babies who are given soy-based formula. Dairy-based formula does not interfere
with thyroxine. Frequent checks must be done to assess the baby for inadequate growth. To
ensure adequate growth and development, the hormone must be taken throughout life.
Page Ref: 812
Cognitive Level: Applying
Client Need: Physiological Integrity
Nursing Process: Planning
Learning Outcome: 6. Plan evidence-based care for an individual with thyroid disease and his or
her family in collaboration with other members of the healthcare team.

62
Copyright © 2015 Pearson Education, Inc.
6) A client with new-onset atrial fibrillation appears very anxious. After reviewing the client's
recent laboratory results, the nurse concludes that what might be causing the client's symptoms?
A) A Hgb of 11.0 g/dL
B) A TSH of 0.25 mU/mL
C) A TSH of 18 mU/mL
D) A Hgb of 13.8 g/dL
Answer: C
Explanation: A) New-onset atrial fibrillation and anxiety are potential symptoms of
hyperthyroidism. A TSH level above 5 mU/mL is considered high. The nurse can plan outcomes
to relieve the client's anxiety based on this diagnosis. TSH 0.25 mU/mL is indicative of
hypothyroidism. Hgb 13.8 g/dL and Hgb 11.0 g/dL are both normal hemoglobin levels.
B) New-onset atrial fibrillation and anxiety are potential symptoms of hyperthyroidism. A TSH
level above 5 mU/mL is considered high. The nurse can plan outcomes to relieve the client's
anxiety based on this diagnosis. TSH 0.25 mU/mL is indicative of hypothyroidism. Hgb 13.8
g/dL and Hgb 11.0 g/dL are both normal hemoglobin levels.
C) New-onset atrial fibrillation and anxiety are potential symptoms of hyperthyroidism. A TSH
level above 5 mU/mL is considered high. The nurse can plan outcomes to relieve the client's
anxiety based on this diagnosis. TSH 0.25 mU/mL is indicative of hypothyroidism. Hgb 13.8
g/dL and Hgb 11.0 g/dL are both normal hemoglobin levels.
D) New-onset atrial fibrillation and anxiety are potential symptoms of hyperthyroidism. A TSH
level above 5 mU/mL is considered high. The nurse can plan outcomes to relieve the client's
anxiety based on this diagnosis. TSH 0.25 mU/mL is indicative of hypothyroidism. Hgb 13.8
g/dL and Hgb 11.0 g/dL are both normal hemoglobin levels.
Page Ref: 807, 809
Cognitive Level: Analyzing
Client Need: Physiological Integrity
Nursing Process: Evaluation
Learning Outcome: 7. Evaluate expected outcomes for an individual with thyroid disease.

63
Copyright © 2015 Pearson Education, Inc.
7) An older female client with hyperthyroidism is not a candidate for surgery. Which treatment
could be used to quickly reduce the client's manifestations of the disorder?
A) Nothing, because there is little effect on the quality of life in older adults.
B) A partial thyroidectomy
C) The ingestion of radioactive sodium iodine, I131
D) A combination treatment with levothyroid (Synthroid) and amiodarone (Cordarone)
Answer: C
Explanation: A) Hyperthyroidism in older clients is most often caused by Graves disease or
toxic nodular goiter, which is diagnosed by subnormal or undetectable levels of TSH (thyroid-
stimulating hormone). The treatment of choice is the ingestion of radioactive iodine, which is
picked up by the thyroid tissue and then destroys the tissue. This treatment avoids surgery,
anesthesia, and hospitalization. Surgical removal is reserved for clients with symptoms too
severe for treatment with radioactive iodine or nodules suspicious for malignancy. Treatment
with levothyroid (Synthroid) would worsen symptoms, and amiodarone (Cordarone) is an
antiarrhythmic drug that has induced hyperthyroidism in some clients. Older clients with
hyperthyroidism seldom present with the classic symptoms seen in younger clients, but older
clients may develop cardiac dysrhythmias, weight loss, fatigue, and apathy, and do require
treatment. A partial thyroidectomy is a surgical procedure, for which the client is not a candidate.
B) Hyperthyroidism in older clients is most often caused by Graves disease or toxic nodular
goiter, which is diagnosed by subnormal or undetectable levels of TSH (thyroid-stimulating
hormone). The treatment of choice is the ingestion of radioactive iodine, which is picked up by
the thyroid tissue and then destroys the tissue. This treatment avoids surgery, anesthesia, and
hospitalization. Surgical removal is reserved for clients with symptoms too severe for treatment
with radioactive iodine or nodules suspicious for malignancy. Treatment with levothyroid
(Synthroid) would worsen symptoms, and amiodarone (Cordarone) is an antiarrhythmic drug that
has induced hyperthyroidism in some clients. Older clients with hyperthyroidism seldom present
with the classic symptoms seen in younger clients, but older clients may develop cardiac
dysrhythmias, weight loss, fatigue, and apathy, and do require treatment. A partial thyroidectomy
is a surgical procedure, for which the client is not a candidate.
C) Hyperthyroidism in older clients is most often caused by Graves disease or toxic nodular
goiter, which is diagnosed by subnormal or undetectable levels of TSH (thyroid-stimulating
hormone). The treatment of choice is the ingestion of radioactive iodine, which is picked up by
the thyroid tissue and then destroys the tissue. This treatment avoids surgery, anesthesia, and
hospitalization. Surgical removal is reserved for clients with symptoms too severe for treatment
with radioactive iodine or nodules suspicious for malignancy. Treatment with levothyroid
(Synthroid) would worsen symptoms, and amiodarone (Cordarone) is an antiarrhythmic drug that
has induced hyperthyroidism in some clients. Older clients with hyperthyroidism seldom present
with the classic symptoms seen in younger clients, but older clients may develop cardiac
dysrhythmias, weight loss, fatigue, and apathy, and do require treatment. A partial thyroidectomy
is a surgical procedure, for which the client is not a candidate.

64
Copyright © 2015 Pearson Education, Inc.
D) Hyperthyroidism in older clients is most often caused by Graves disease or toxic nodular
goiter, which is diagnosed by subnormal or undetectable levels of TSH (thyroid-stimulating
hormone). The treatment of choice is the ingestion of radioactive iodine, which is picked up by
the thyroid tissue and then destroys the tissue. This treatment avoids surgery, anesthesia, and
hospitalization. Surgical removal is reserved for clients with symptoms too severe for treatment
with radioactive iodine or nodules suspicious for malignancy. Treatment with levothyroid
(Synthroid) would worsen symptoms, and amiodarone (Cordarone) is an antiarrhythmic drug that
has induced hyperthyroidism in some clients. Older clients with hyperthyroidism seldom present
with the classic symptoms seen in younger clients, but older clients may develop cardiac
dysrhythmias, weight loss, fatigue, and apathy, and do require treatment. A partial thyroidectomy
is a surgical procedure, for which the client is not a candidate.
Page Ref: 809
Cognitive Level: Analyzing
Client Need: Physiological Integrity
Nursing Process: Planning
Learning Outcome: 5. Summarize therapies used by interdisciplinary teams in the collaborative
care of an individual with thyroid disease.

65
Copyright © 2015 Pearson Education, Inc.
8) The nurse is planning to teach food-drug interactions for a client who was recently diagnosed
with hyperthyroidism. Which food should the nurse teach the client to avoid?
A) Caffeine-free soda
B) Soy sauce
C) Milk products
D) High-calorie foods
Answer: B
Explanation: A) Foods high in iodine, such as soy sauce, can influence the effectiveness of
medication therapy for clients who are diagnosed with hyperthyroidism. High-calorie foods are
important for clients with hyperthyroidism in order to meet metabolic demands. There is no
reason to restrict caffeine-free soda. Milk products should be included in the diet for the client
with hyperthyroidism because they are high in protein and calcium.
B) Foods high in iodine, such as soy sauce, can influence the effectiveness of medication therapy
for clients who are diagnosed with hyperthyroidism. High-calorie foods are important for clients
with hyperthyroidism in order to meet metabolic demands. There is no reason to restrict caffeine-
free soda. Milk products should be included in the diet for the client with hyperthyroidism
because they are high in protein and calcium.
C) Foods high in iodine, such as soy sauce, can influence the effectiveness of medication therapy
for clients who are diagnosed with hyperthyroidism. High-calorie foods are important for clients
with hyperthyroidism in order to meet metabolic demands. There is no reason to restrict caffeine-
free soda. Milk products should be included in the diet for the client with hyperthyroidism
because they are high in protein and calcium.
D) Foods high in iodine, such as soy sauce, can influence the effectiveness of medication therapy
for clients who are diagnosed with hyperthyroidism. High-calorie foods are important for clients
with hyperthyroidism in order to meet metabolic demands. There is no reason to restrict caffeine-
free soda. Milk products should be included in the diet for the client with hyperthyroidism
because they are high in protein and calcium.
Page Ref: 812
Cognitive Level: Applying
Client Need: Health Promotion and Maintenance
Nursing Process: Implementation
Learning Outcome: 6. Plan evidence-based care for an individual with thyroid disease and his or
her family in collaboration with other members of the healthcare team.

66
Copyright © 2015 Pearson Education, Inc.
9) A client with exophthalmos is pictured below. The nurse understands that:

A) This condition is caused by hypothyroidism.


B) Treatment for this condition reverses the symptoms of exophthalmos.
C) This condition is caused by Hashimoto thyroiditis.
D) People with this condition usually have very low blood pressure.
E) Women with this condition may have fertility problems.
Answer: E
Explanation: A) Exophthalmos is a condition associated with Graves disease, caused by
hyperthyroidism. Treatment for this condition may stabilize the symptoms but does not reverse
them. Hashimoto thyroiditis is condition of hypothyroidism, not hyperthyroidism. People with
exophthalmos and Graves disease usually have hypertension, not hypotension. Women with this
condition may have problems with fertility.
B) Exophthalmos is a condition associated with Graves disease, caused by hyperthyroidism.
Treatment for this condition may stabilize the symptoms but does not reverse them. Hashimoto
thyroiditis is condition of hypothyroidism, not hyperthyroidism. People with exophthalmos and
Graves disease usually have hypertension, not hypotension. Women with this condition may
have problems with fertility.
C) Exophthalmos is a condition associated with Graves disease, caused by hyperthyroidism.
Treatment for this condition may stabilize the symptoms but does not reverse them. Hashimoto
thyroiditis is condition of hypothyroidism, not hyperthyroidism. People with exophthalmos and
Graves disease usually have hypertension, not hypotension. Women with this condition may
have problems with fertility.
D) Exophthalmos is a condition associated with Graves disease, caused by hyperthyroidism.
Treatment for this condition may stabilize the symptoms but does not reverse them. Hashimoto
thyroiditis is condition of hypothyroidism, not hyperthyroidism. People with exophthalmos and
Graves disease usually have hypertension, not hypotension. Women with this condition may
have problems with fertility.
E) Exophthalmos is a condition associated with Graves disease, caused by hyperthyroidism.
Treatment for this condition may stabilize the symptoms but does not reverse them. Hashimoto
thyroiditis is condition of hypothyroidism, not hyperthyroidism. People with exophthalmos and
Graves disease usually have hypertension, not hypotension. Women with this condition may
have problems with fertility.
Page Ref: 806
Cognitive Level: Applying
Client Need: Physiological Integrity
Nursing Process: Assessment
Learning Outcome: 1. Describe the pathophysiology, etiology, clinical manifestations, and
direct and indirect causes of thyroid disease.
67
Copyright © 2015 Pearson Education, Inc.
10) A client with Graves disease requests that the nurse discuss the laboratory tests of the client.
Which statements would the nurse include?
Select all that apply.
A) "Your TSH level is increased."
B) "Your thyroid antibodies test is increased."
C) "Your serum T4 is decreased."
D) "Your serum T3 is increased."
E) "Your T3 uptake is decreased."
Answer: B, D
Explanation: A) Graves disease, or primary hyperthyroidism, has alterations in normal lab work.
With this condition, TSH levels are decreased. Thyroid antibodies, serum T4, serum T3, and T3
uptake tests are all increased.
B) Graves disease, or primary hyperthyroidism, has alterations in normal lab work. With this
condition, TSH levels are decreased. Thyroid antibodies, serum T4, serum T3, and T3 uptake
tests are all increased.
C) Graves disease, or primary hyperthyroidism, has alterations in normal lab work. With this
condition, TSH levels are decreased. Thyroid antibodies, serum T4, serum T3, and T3 uptake
tests are all increased.
D) Graves disease, or primary hyperthyroidism, has alterations in normal lab work. With this
condition, TSH levels are decreased. Thyroid antibodies, serum T4, serum T3, and T3 uptake
tests are all increased.
E) Graves disease, or primary hyperthyroidism, has alterations in normal lab work. With this
condition, TSH levels are decreased. Thyroid antibodies, serum T4, serum T3, and T3 uptake
tests are all increased.
Page Ref: 809
Cognitive Level: Applying
Client Need: Physiological Integrity
Nursing Process: Evaluation
Learning Outcome: 5. Summarize therapies used by interdisciplinary teams in the collaborative
care of an individual with thyroid disease.

68
Copyright © 2015 Pearson Education, Inc.

You might also like